+7 495 120-13-73 | 8 800 500-97-74

(для регионов бесплатно)

Содержание

Конспект урока по физике «применение закона джоуля-ленца»

Определения

В словесной формулировке звучит следующим образом:

Математически может быть выражен в следующей форме:

где  — мощность выделения тепла в единице объёма,  — плотность электрического тока,  — напряжённость электрического поля, σ — проводимость среды, а точкой обозначено скалярное произведение.

Закон также может быть сформулирован в интегральной форме для случая протекания токов в тонких проводах:

В интегральной форме этот закон имеет вид

где  — количество теплоты, выделяемое за промежуток времени ,  — сила тока,  — сопротивление,  — полное количество теплоты, выделенное за промежуток времени от до . В случае постоянных силы тока и сопротивления:

Применяя закон Ома, можно получить следующие эквивалентные формулы:

Свойства электрического тока

Когда электрический ток проходит через металлический проводник, его электроны постоянно сталкиваются с различными посторонними частицами.

Это могут быть обычные нейтральные молекулы или молекулы, потерявшие электроны. Электрон в процессе движения может отщепить от нейтральной молекулы еще один электрон. В результате, его кинетическая энергия теряется, а вместо молекулы происходит образование положительного иона. В других случаях электрон, наоборот, соединиться с положительным ионом и образовать нейтральную молекулу.

В процессе столкновений электронов и молекул происходит расход энергии, в дальнейшем превращающейся в тепло. Затраты определенного количества энергии связаны со всеми движениями, во время которых приходится преодолевать сопротивление. В это время происходит превращение работы, затраченной на преодоление сопротивления трения, в тепловую энергию.

Опыты, демонстрирующие зависимость количества теплоты от силы тока и сопротивления

Факт нагрева проводника при протекании по нему тока объясняется тем, что во время движения заряженных частиц под действием электрического поля они сталкиваются с частицами проводника, в результате часть энергии передаётся этим частицам проводника, то есть средняя скорость хаотического (теплового) движения частиц проводника увеличивается, и проводник нагревается. По закону сохранения энергии кинетическая энергия свободных заряженных частиц, приобретённая под действием электрического поля, превратится во внутреннюю энергию проводника. Следовательно, можно предположить:

1. чем больше сопротивление проводника, тем больше тепла выделяется при прохождении электрического тока по проводнику, то есть количество теплоты, которое выделяется в проводнике при прохождении по нему электрического тока, прямо пропорционально сопротивлению проводника;

2. количество теплоты, выделяемое в проводнике при прохождении по нему электрического тока, зависит от силы тока (чем больше сила тока, тем большее количество свободных частиц проходит через сечение проводника в единицу времени, происходит больше столкновений, следовательно, больше энергии передаётся частицам проводника). Можно подтвердить данные предположения с помощью опытов.

Соберём электрическую цепь, в которой последовательно с источником тока подключены два нагревателя с разными сопротивлениями, которые опущены в калориметры (прибор для измерения количества теплоты) с одинаковым количеством воды при одинаковой температуре. При прохождении электрического тока через нагреватели будет наблюдаться повышение температуры воды, причём вода будет нагреваться быстрее в том калориметре, в который помещён нагреватель с бльшим сопротивлением (см. Рис. 1). То есть подтверждается предположение 1.

Для подтверждения предположения 2 соберём электрическую цепь, в которой последовательно к источнику тока подключен амперметр, лампочка накаливания и реостат. Регулируя сопротивление реостата, меняем силу тока в цепи при постоянном напряжении. При увеличении силы тока увеличивается яркость лампочки (см. Рис. 2), то есть увеличивается количество теплоты, которое выделяет нить накаливания.

Рис. 1. Нагреватель с бльшим сопротивлением нагревает воду быстрее

Рис. 2. Увеличение яркости лампочки при увеличении силы тока    

Где может пригодиться этот закон Джоуля-Ленца?

В электротехнике есть понятие длительно допустимого тока протекающего по проводам. Это такой ток, который провод способен выдержать длительное время (то есть, бесконечно долго), без разрушения провода (и изоляции, если она есть, потому что провод может быть и без изоляции).

Конечно, данные вы теперь можете взять из ПУЭ (Правила устройства электроустановок), но получали эти данные исключительно на основе закона Джоуля-Ленца.

В электротехнике так же используются плавкие предохранители. Их основное качество – надёжность срабатывания. Для этого используется проводник определенного сечения. Зная температуру плавления такого проводника можно вычислить количество теплоты, которое необходимо, чтобы проводник расплавился от протекания через него больших значений тока, а вычислив ток, можно вычислить и сопротивление, которым такой проводник должен обладать. В общем, как вы уже поняли, применяя закон Джоуля-Ленца можно рассчитать сечение или сопротивление (величины взаимозависимы) проводника для плавкого предохранителя.

А ещё, помните, мы говорили про . Там на примере лампочки я рассказывал парадокс, что более мощная лампа в последовательном соединении светит слабее. И наверняка помните почему: падение напряжения на сопротивлении тем сильнее, чем меньше сопротивление. А поскольку мощность — это , а напряжение очень сильно падает, то и выходит, что большое сопротивление выделит большое количество тепла, то есть, току придется больше потрудиться, чтобы преодолеть большое сопротивление. И количество тепла, которое выделит ток при этом можно посчитать с помощью закона Джоуля-Ленца. Если брать последовательное соединение сопротивлений, то использовать лучше выражение через квадрат тока, то есть, изначальный вид формулы:

А для параллельного соединения сопротивлений, поскольку ток в параллельных ветвях зависит от сопротивления, в то время, как напряжение на каждой параллельной ветви одинаковое, то формулу лучше всего представить через напряжение:

Примерами работы закона Джоуля-Ленца вы все пользуетесь в повседневной жизни – в первую очередь это всевозможные нагревательные приборы. Как правило, в них используется нихромовая проволока и толщина (поперечное сечение) и длина проводника подбираются с учётом того, чтобы длительное тепловое воздействие не приводило к стремительному разрушению проволоки.

Точно таким же образом добиваются свечения вольфрамовой нити в лампе накаливания. По этому же закону определяют степень возможного нагрева практически любого электротехнического и электронного устройства.

В общем, несмотря на кажущуюся простоту, закон Джоуля-Ленца играет в нашей жизни очень огромную роль. Этот закон дал большой толчок для теоретических расчётов: выделение тепла токами , вычисление конкретной температуры дуги, проводника и любого другого электропроводного материала, потери электрической мощности в тепловом эквиваленте и т.д.

Вы можете спросить, а как перевести Джоули в Ватты и это довольно частый вопрос в интернете. Хотя вопрос несколько неверный, читая далее, вы поймёте почему. Ответ довольно прост: 1 дж = 0.000278 Ватт*час, в то время, как 1 Ватт*час = 3600 Джоулей. Напомню, что в Ваттах измеряется потребляемая мгновенная мощность, то есть непосредственно используемая пока включена цепь. А Джоуль определяет работу электрического тока, то есть мощность тока за промежуток времени. Помните, в законе Ома я приводил аллегорическую ситуацию. Ток – деньги, напряжение – магазин, сопротивление – чувство меры и денег, мощность – количество продуктов, которые вы сможете на себе унести (увезти) за один раз, а вот как далеко, как быстро и сколько раз вы сможете их увезти – это работа. То есть, сравнить работу и мощность никак не получается, но можно выразить в более понятных нам единицам: Ваттах и часах.

Думаю, что теперь вам не составит труда применить закон Джоуля-Ленца в практике и теории, если таковое потребуется и даже сделать перевод Джоулей в Ватты и наоборот. А благодаря пониманию, что закон Джоуля-Ленца это произведение электрической мощности на время, вы сможете более легко его запомнить и даже, если вдруг забыли основную формулу, то помня всего лишь закон Ома можно снова получить закон Джоуля-Ленца. А я на этом с вами прощаюсь.

Знаменитый русский физик Ленц и английский физик Джоуль, проводя опыты по изучению тепловых действий электрического тока, независимо друг от друга вывели закон Джоуля-Ленца.

Данный закон отражает взаимосвязь количества теплоты, выделяемого в проводнике, и электрического тока, проходящего по этому проводнику в течение определенного периода времени.

Что такое ток

Ток – это упорядоченное движение заряженных частиц, которые называются электронами. И если ток протекает по проводнику, то в нём начинают происходить разные физические процессы, а именно сталкиваются электроны с молекулами.

Молекулы бывают нейтральные или те, которые потеряли свою отрицательно заряженную частицу. В результате столкновений или электроны могут становиться нейтральными молекулами, или при этом выбивается из другой такой же молекулы электрон, образовавший положительно заряженный ион. Во время этих столкновений расходуется кинетическая энергия заряженных частиц. Именно эта энергия и становится теплом.

На тепловой нагрев проводника может влиять и сопротивление. Например, можно взять определённое тело и тащить его по земле. Земля в этом случае — сопротивление. Что же с ним будет? Правильно, между телом и поверхностью будет происходить сила трения, которая, в свою очередь, нагревает тело. Ток в этом случае ведёт себя точно так же.

Практическое значение

Снижение потерь энергии


При передаче электроэнергии тепловое действие тока является нежелательным, поскольку ведёт к потерям энергии. Поскольку передаваемая мощность линейно зависит как от напряжения, так и от силы тока, а мощность нагрева зависит от силы тока квадратично, то выгодно повышать напряжение перед передачей электроэнергии , понижая в результате силу тока. Однако, повышение напряжения снижает электробезопасность линий электропередачи .

Для применения высокого напряжения в цепи для сохранения прежней мощности на полезной нагрузке приходится увеличивать сопротивление нагрузки. Подводящие провода и нагрузка соединены последовательно . Сопротивление проводов () можно считать постоянным. А вот сопротивление нагрузки () растёт при выборе более высокого напряжения в сети. Также растёт соотношение сопротивления нагрузки и сопротивления проводов. При последовательном включении сопротивлений (провод — нагрузка — провод) распределение выделяемой мощности () пропорционально сопротивлению подключённых сопротивлений.

Ток в сети для всех сопротивлений постоянен. Следовательно, выполняются соотношение

И для в каждом конкретном случае являются константами. Следовательно, мощность, выделяемая на проводах, обратно пропорциональна сопротивлению нагрузки, то есть уменьшается с ростом напряжения, так как . Откуда следует, что . В каждом конкретном случае величина является константой, следовательно, тепло выделяемое на проводе обратно пропорционально квадрату напряжения на потребителе.

Выбор проводов для цепей


Тепло, выделяемое проводником с током, в той или иной степени выделяется в окружающую среду. В случае, если сила тока в выбранном проводнике превысит некоторое предельно допустимое значение, возможен столь сильный нагрев, что проводник может спровоцировать возгорание находящихся рядом с ним объектов или расплавиться сам. Как правило, при сборке электрических цепей достаточно следовать принятым нормативным документам, которые регламентируют, в частности, выбор сечения проводников.

Электронагревательные приборы


Если сила тока одна и та же на всём протяжении электрической цепи, то в любом выбранном участке будет выделять тепла тем больше, чем выше сопротивление данного участка.

За счёт сознательного увеличения сопротивления участка цепи можно добиться локализованного выделения тепла в этом участке. По этому принципу работают электронагревательные приборы
. В них используется нагревательный элемент
— проводник с высоким сопротивлением. Повышение сопротивления достигается (совместно или по отдельности) выбором сплава с высоким удельным сопротивлением (например, нихром , константан), увеличением длины проводника и уменьшением его поперечного сечения. Подводящие провода имеют обычное низкое сопротивление и поэтому их нагрев, как правило, незаметен.

Плавкие предохранители


Для защиты электрических цепей от протекания чрезмерно больших токов используется отрезок проводника со специальными характеристиками. Это проводник относительно малого сечения и из такого сплава, что при допустимых токах нагрев проводника не перегревает его, а при чрезмерно больших перегрев проводника столь значителен, что проводник расплавляется и размыкает цепь.

Теплота и энергия в электрической цепи

Процесс преобразования электрической энергии в тепловую играет большую роль в практическом применении, что широко используется в разных нагревательных приборах в промышленной и бытовой сфере.

В то же время, тепловые потери нежелательны по причине того, что могут сопровождаться непроизводительными расходами энергии. Это может касаться, например, электрических машин, трансформаторов и прочих устройств, что существенно снижает их КПД.

Закон Джоуля-Ленца

Замечание 1

Первым сформулировал зависимость выделения теплоты от силы электрического тока Джеймс Джоуль, что произошло в 1841 году. Позднее это сделал Эмиль Ленц. Так появляется закон Джоуля-Ленца, позволяющий рассчитывать мощность электронагревателей наряду с потерями на тепловыделение в линиях электропередач. 2t}{R} = UIt$

Исследования физиков Джоуля и Ленца относительно тепловыделения от действия электрического тока значительно продвинули научное понимание определенных физических процессов, а выведенные при этом основные формулы, не претерпев изменений, продолжают активно использоваться в различных научно-технических отраслях.

В сфере электротехники выделяют несколько технических задач, где количество теплоты, которая будет выделяться при протекании тока, имеет критически важное значение при расчете таких параметров, как:

  • теплопотери в ЛЭП;
  • характеристики для проводов сетей электропроводки;
  • тепловая мощность электронагревателей;
  • температура срабатывания автовыключателей;
  • температура плавления плавких предохранителей;
  • тепловыделение разных электротехнических аппаратов, а также элементов радиотехники.

Замечание 2

Тепловое действие электротока в проводах ЛЭП является нежелательным из-за весомых потерь электроэнергии на тепловое выделение. 2}{R}$

В системе СИ энергия и мощность измеряются в Джоулях (Дж) и Ваттах (Вт) соответственно. Для всех приведенных выше величин применяются кратные и дольные единицы измерения. Энергию часто выражают в киловатт-часах.

ЭДС. Закон Ома для полной цепи

Автор статьи — профессиональный репетитор, автор учебных пособий для подготовки к ЕГЭ Игорь Вячеславович Яковлев

Темы кодификатора ЕГЭ: электродвижущая сила, внутреннее сопротивление источника тока, закон Ома для полной электрической цепи.

До сих пор при изучении электрического тока мы рассматривали направленное движение свободных зарядов во внешней цепи, то есть в проводниках, подсоединённых к клеммам источника тока.

Как мы знаем, положительный заряд :

• уходит во внешнюю цепь с положительной клеммы источника;

• перемещается во внешней цепи под действием стационарного электрического поля, создаваемого другими движущимися зарядами;

• приходит на отрицательную клемму источника, завершая свой путь во внешней цепи.

Теперь нашему положительному заряду нужно замкнуть свою траекторию и вернуться на положительную клемму. Для этого ему требуется преодолеть заключительный отрезок пути — внутри источника тока от отрицательной клеммы к положительной. Но вдумайтесь: идти туда ему совсем не хочется! Отрицательная клемма притягивает его к себе, положительная клемма его от себя отталкивает, и в результате на наш заряд внутри источника действует электрическая сила , направленная против движения заряда (т.е. против направления тока).

Сторонняя сила

Тем не менее, ток по цепи идёт; стало быть, имеется сила, «протаскивающая» заряд сквозь источник вопреки противодействию электрического поля клемм (рис. 1).

Рис. 1. Сторонняя сила

Эта сила называется сторонней силой; именно благодаря ей и функционирует источник тока. Сторонняя сила не имеет отношения к стационарному электрическому полю — у неё, как говорят, неэлектрическое происхождение; в батарейках, например, она возникает благодаря протеканию соответствующих химических реакций.

Обозначим через работу сторонней силы по перемещению положительного заряда q внутри источника тока от отрицательной клеммы к положительной. Эта работа положительна, так как направление сторонней силы совпадает с направлением перемещения заряда. Работа сторонней силы называется также работой источника тока.

Во внешней цепи сторонняя сила отсутствует, так что работа сторонней силы по перемещению заряда во внешней цепи равна нулю. Поэтому работа сторонней силы по перемещению заряда вокруг всей цепи сводится к работе по перемещению этого заряда только лишь внутри источника тока. Таким образом, — это также работа сторонней силы по перемещению заряда по всей цепи.

Мы видим, что сторонняя сила является непотенциальной — её работа при перемещении заряда по замкнутому пути не равна нулю. Именно эта непотенциальность и обеспечивает циркулирование электрического тока; потенциальное электрическое поле, как мы уже говорили ранее, не может поддерживать постоянный ток.

Опыт показывает, что работа прямо пропорциональна перемещаемому заряду . Поэтому отношение уже не зависит от заряда и является количественной характеристикой источника тока. Это отношение обозначается :

(1)

Данная величина называется электродвижущей силой (ЭДС) источника тока. Как видим, ЭДС измеряется в вольтах (В), поэтому название «электродвижущая сила» является крайне неудачным. Но оно давно укоренилось, так что приходится смириться.

Когда вы видите надпись на батарейке: «1,5 В», то знайте, что это именно ЭДС. Равна ли эта величина напряжению, которое создаёт батарейка во внешней цепи? Оказывается, нет! Сейчас мы поймём, почему.

Закон Ома для полной цепи

Любой источник тока обладает своим сопротивлением , которое называется внутренним сопротивлением этого источника. Таким образом, источник тока имеет две важных характеристики: ЭДС и внутреннее сопротивление.

Пусть источник тока с ЭДС, равной , и внутренним сопротивлением подключён к резистору (который в данном случае называется внешним резистором, или внешней нагрузкой, или полезной нагрузкой). Всё это вместе называется полной цепью (рис. 2).

Рис. 2. Полная цепь

Наша задача — найти силу тока в цепи и напряжение на резисторе .

За время по цепи проходит заряд . Согласно формуле (1) источник тока совершает при этом работу:

(2)

Так как сила тока постоянна, работа источника целиком превращается в теплоту, которая выделяется на сопротивлениях и . Данное количество теплоты определяется законом Джоуля–Ленца:

(3)

Итак, , и мы приравниваем правые части формул (2) и (3):

После сокращения на получаем:

Вот мы и нашли ток в цепи:

(4)

Формула (4) называется законом Ома для полной цепи.

Если соединить клеммы источника проводом пренебрежимо малого сопротивления , то получится короткое замыкание. Через источник при этом потечёт максимальный ток — ток короткого замыкания:

Из-за малости внутреннего сопротивления ток короткого замыкания может быть весьма большим. Например, пальчиковая батарейка разогревается при этом так, что обжигает руки.

Зная силу тока (формула (4)), мы можем найти напряжение на резисторе с помощью закона Ома для участка цепи:

(5)

Это напряжение является разностью потенциалов между точками и (рис. 2). Потенциал точки равен потенциалу положительной клеммы источника; потенциал точки равен потенциалу отрицательной клеммы. Поэтому напряжение (5) называется также напряжением на клеммах источника.

Мы видим из формулы (5), что в реальной цепи будет — ведь умножается на дробь, меньшую единицы. Но есть два случая, когда .

1. Идеальный источник тока. Так называется источник с нулевым внутренним сопротивлением. При формула (5) даёт .

2. Разомкнутая цепь. Рассмотрим источник тока сам по себе, вне электрической цепи. В этом случае можно считать, что внешнее сопротивление бесконечно велико: . Тогда величина неотличима от , и формула (5) снова даёт нам .

Смысл этого результата прост: если источник не подключён к цепи, то вольтметр, подсоединённый к полюсам источника, покажет его ЭДС.

КПД электрической цепи

Нетрудно понять, почему резистор называется полезной нагрузкой. Представьте себе, что это лампочка. Теплота, выделяющаяся на лампочке, является полезной, так как благодаря этой теплоте лампочка выполняет своё предназначение — даёт свет.

Количество теплоты, выделяющееся на полезной нагрузке за время , обозначим .

Если сила тока в цепи равна , то

Некоторое количество теплоты выделяется также на источнике тока:

Полное количество теплоты, которое выделяется в цепи, равно:

КПД электрической цепи — это отношение полезного тепла к полному:

КПД цепи равен единице лишь в том случае, если источник тока идеальный .

Закон Ома для неоднородного участка

Простой закон Ома справедлив для так называемого однородного участка цепи — то есть участка, на котором нет источников тока. Сейчас мы получим более общие соотношения, из которых следует как закон Ома для однородного участка, так и полученный выше закон Ома для полной цепи.

Участок цепи называется неоднородным, если на нём имеется источник тока. Иными словами, неоднородный участок — это участок с ЭДС.

На рис. 3показан неоднородный участок, содержащий резистор и источник тока. ЭДС источника равна , его внутреннее сопротивление считаем равным нулю (усли внутреннее сопротивление источника равно , можно просто заменить резистор на резистор ).

Рис. 3. ЭДС «помогает» току:

Сила тока на участке равна , ток течёт от точки к точке . Этот ток не обязательно вызван одним лишь источником . Рассматриваемый участок, как правило, входит в состав некоторой цепи (не изображённой на рисунке), а в этой цепи могут присутствовать и другие источники тока. Поэтому ток является результатом совокупного действия всех источников, имеющихся в цепи.

Пусть потенциалы точек и равны соответственно и . Подчеркнём ещё раз, что речь идёт о потенциале стационарного электрического поля, порождённого действием всех источников цепи — не только источника, принадлежащего данному участку, но и, возможно, имеющихся вне этого участка.

Напряжение на нашем участке равно: . За время через участок проходит заряд , при этом стационарное электрическое поле совершает работу:

Кроме того, положительную работу совершает источник тока (ведь заряд прошёл сквозь него!):

Сила тока постоянна, поэтому суммарная работа по продвижению заряда , совершаемая на участке стационарным элетрическим полем и сторонними силами источника, целиком превращается в тепло: .

Подставляем сюда выражения для , и закон Джоуля–Ленца:

Сокращая на , получаем закон Ома для неоднородного участка цепи:

(6)

или, что то же самое:

(7)

Обратите внимание: перед стоит знак «плюс». Причину этого мы уже указывали — источник тока в данном случае совершает положительную работу, «протаскивая» внутри себя заряд от отрицательной клеммы к положительной. Попросту говоря, источник «помогает» току протекать от точки к точке .

Отметим два следствия выведенных формул (6) и (7).

1. Если участок однородный, то . Тогда из формулы (6) получаем — закон Ома для однородного участка цепи.

2. Предположим, что источник тока обладает внутренним сопротивлением . Это, как мы уже упоминали, равносильно замене на :

Теперь замкнём наш участок, соединив точки и . Получим рассмотренную выше полную цепь. При этом окажется, что и предыдущая формула превратится в закон Ома для полной цепи:

Таким образом, закон Ома для однородного участка и закон Ома для полной цепи оба вытекают из закона Ома для неоднородного участка.

Может быть и другой случай подключения, когда источник «мешает» току идти по участку. Такая ситуация изображена на рис. 4. Здесь ток, идущий от к , направлен против действия сторонних сил источника.

Рис. 4. ЭДС «мешает» току:

Как такое возможно? Очень просто: другие источники, имеющиеся в цепи вне рассматриваемого участка, «пересиливают» источник на участке и вынуждают ток течь против . Именно так происходит, когда вы ставите телефон на зарядку: подключённый к розетке адаптер вызывает движение зарядов против действия сторонних сил аккумулятора телефона, и аккумулятор тем самым заряжается!

Что изменится теперь в выводе наших формул? Только одно — работа сторонних сил станет отрицательной:

Тогда закон Ома для неоднородного участка примет вид:

(8)

или:

где по-прежнему — напряжение на участке.

Давайте соберём вместе формулы (7) и (8) и запишем закон Ома для участка с ЭДС следующим образом:

Ток при этом течёт от точки к точке . Если направление тока совпадает с направлением сторонних сил, то перед ставится «плюс»; если же эти направления противоположны, то ставится «минус».

4.4.4. Эффективные напряжение и ток

Силу переменного тока (напряжения) можно охарактеризовать при помощи амплитуды. Однако амплитудное значение тока непросто измерить экспериментально. Силу переменного тока удобно связать с каким-либо действием, производимым током, не зависящим от его направления. Таковым является, например, тепловое действие тока. Поворот стрелки амперметра, измеряющего переменный ток, вызывается удлинением нити, которая нагревается при прохождении по ней тока.

Действующим илиэффективнымзначением переменного тока (напряжения) называется такое значение постоянного тока, при котором на активном сопротивлении выделяется за период такое же количество теплоты, как и при переменном токе.

Свяжем эффективное значение тока с его амплитудным значением. Для этого рассчитаем количество теплоты, выделяемое на активном сопротивлении переменным током за время, равное периоду колебаний. Напомним, что по закону Джоуля-Ленца количество теплоты, выделяющееся на участке цепи cсопротивлениемприпостоянномтокеза время, определяется по формуле. Переменный ток можно считать постоянным только в течение очень малых промежутков времени. Поделим период колебанийна очень большое число малых промежутков времени. Количество теплоты, выделяемое на сопротивленииза время:. Общее количество теплоты, выделяемое за период, найдется суммированием теплот, выделяемых за отдельные малые промежутки времени, или, другими словами, интегрированием:

.

Сила тока в цепи изменяется по синусоидальному закону

,

тогда

.

Опуская вычисления, связанные с интегрированием, запишем окончательный результат

.

Если бы по цепи шёл некоторый постоянный ток , то за время, равное, выделилось бы тепло. По определению постоянный ток, оказывающий такое же тепловое действие, что и переменный, будет равен эффективному значению переменного тока. Находим эффективное значение силы тока, приравнивая теплоты, выделяемые за период, в случаях постоянного и переменного токов

(4.28)

Очевидно, точно такое же соотношение связывает эффективное и амплитудное значения напряжения в цепи с синусоидальным переменным током:

(4.29)

Например, стандартное напряжение в сети 220 В – это эффективное напряжение. По формуле (4.29) легко посчитать, что амплитудное значение напряжения в этом случае будет равно 311 В.

4.4.5. Мощность в цепи переменного тока

Пусть на некотором участке цепи с переменным током сдвиг фаз между током и напряжением равен , т.е. сила тока и напряжение изменяются по законам:

,.

Тогда мгновенное значение мощности, выделяемой на участке цепи,

.

Мощность изменяется со временем. Поэтому можно говорить лишь о ее среднем значении. Определим среднюю мощность, выделяемую в течение достаточно длительного промежутка времени (во много раз превосходящего период колебаний):

.

С использованием известной тригонометрической формулы

получим

.

Величину усреднять не нужно, так как она не зависит от времени, следовательно:

.

За длительное время значение косинуса много раз успевает измениться, принимая как отрицательные, так и положительные значения в пределах от (1) до 1. Понятно, что среднее во времени значение косинуса равно нулю

, поэтому(4.30)

Выражая амплитуды тока и напряжения через их эффективные значения по формулам (4.28) и (4.29), получим

. (4.31)

Мощность, выделяемая на участке цепи с переменным током, зависит от эффективных значений тока и напряжения и сдвига фаз между током и напряжением. Например, если участок цепи состоит из одного только активного сопротивления, тои. Если участок цепи содержит только индуктивность или только ёмкость, тои.

Объяснить среднее нулевое значение мощности, выделяемой на индуктивности и ёмкости можно следующим образом. Индуктивность и ёмкость лишь заимствуют энергию у генератора, а затем возвращают её обратно. Конденсатор заряжается, а затем разряжается. Сила тока в катушке увеличивается, затем снова спадает до нуля и т. д. Именно по той причине, что на индуктивном и ёмкостном сопротивлениях средняя расходуемая генератором энергия равна нулю, их назвали реактивными. На активном же сопротивлении средняя мощность отлична от нуля. Другими словами провод с сопротивлением при протекании по нему тока нагревается. И энергия, выделяемая в виде тепла, назад в генератор уже не возвращается.

Если участок цепи содержит несколько элементов, то сдвига фаз может быть иным. Например, в случае участка цепи, изображенного на рис. 4.5, сдвиг фаз между током и напряжением определяется по формуле (4.27).

Пример 4.7.К генератору переменного синусоидального тока подключён резистор с сопротивлением. Во сколько раз изменится средняя мощность, расходуемая генератором, если к резистору подключить катушку с индуктивным сопротивлениема) последовательно, б) параллельно (рис. 4.10)? Активным сопротивлением катушки пренебречь.

Решение.Когда к генератору подключено одно только активное сопротивление, расходуемая мощность

(см. формулу (4.30)).

Рассмотрим цепь на рис. 4.10, а. В примере 4.6 было определено амплитудное значение силы тока генератора: . Из векторной диаграммы на рис. 4.11,а определяем сдвиг фаз между током и напряжением генератора

.

В результате средняя расходуемая генератором мощность

.

Ответ: при последовательном включении в цепь индуктивности средняя мощность, расходуемая генератором, уменьшится в 2 раза.

Рассмотрим цепь на рис. 4.10,б. В примере 4.6 было определено амплитудное значение силы тока генератора . Из векторной диаграммы на рис. 4.11,б определяем сдвиг фаз между током и напряжением генератора

.

Тогда средняя мощность, расходуемая генератором

.

Ответ: при параллельном включении индуктивности средняя мощность, расходуемая генератором, не изменяется.

Закон Ома для участка цепи. Закон Джоуля — Ленца. Работа и мощность электрического тока. Виды соединения проводников.

Количество теплоты, выделившееся при прохождении электрического тока по проводнику, прямо пропорционально квадрату силы тока, сопротивлению проводника и времени, в течение которого шел ток: 

Последовательное соединение.

1. Сила тока во всех последовательно соединенных участках цепи одинакова:

I1=I2=I3=…=In=…

2. Напряжение в цепи, состоящей из нескольких последовательно соединенных участков, равно сумме напряжений на каждом участке:

U=U1+U2+…+Un+…

3. Сопротивление цепи, состоящей из нескольких последовательно соединенных участков, равно сумме сопротивлений каждого участка:

R=R1+R2+…+Rn+…

Если все сопротивления в цепи одинаковы, то:

R=R1. N

При последовательном соединении общее сопротивление увеличивается (больше большего).

Параллельное соединение.

1. Сила тока в неразветвленном участке цепи равна сумме сил токов во всех параллельно соединенных участках.

I=I1+I2+…+In+…

2. Напряжение на всех параллельно соединенных участках цепи одинаково:    

U1=U2=U3=…=Un=…

 3. При параллельном соединении проводников проводимости складываются (складываются величины, обратные сопротивлению):

Если все сопротивления в цепи одинаковы, то: 

При параллельном соединении общее сопротивление уменьшается (меньше меньшего).

4. Работа электрического тока в цепи, состоящей из последовательно соединенных участков, равна сумме работ на отдельных участках:

A=A1+A2+…+An+…  

т.к.  A=I2Rt=I2(R1+R2+…+Rn+…)t.

5. Мощность электрического тока в цепи, состоящей из последовательно соединенных участков, равна сумме мощностей на отдельных участках:

P=P1+P2+…+Pn+…  

6. Т.к. силы тока во всех участках одинаковы, то:       U1:U2:…:Un:…  = R1:R2:…:Rn:…

Для двух резисторов:  — чем больше сопротивление, тем больше напряжение.

4. Работа электрического тока в цепи, состоящей из параллельно соединенных участков, равна сумме работ на отдельных участках:

A=A1+A2+…+An+…   

т.к.     .

 

5. Мощность электрического тока в цепи, состоящей из параллельно соединенных участков, равна сумме мощностей на отдельных участках:

P=P1+P2+…+Pn+…  

6. Т.к. напряжения на всех участках одинаковы, то:

I1R1= I2R2=…= I3R3=…

Для двух резисторов:  — чем больше сопротивление, тем меньше сила тока.

Что такое модуль Пельтье

В основе работы модуля Пельтье лежит следующий эффект:

при протекании постоянного электрического тока в цепи, состоящей из разнородных проводников, в местах контактов проводников поглощается или выделяется, в зависимости от направления тока, теплота. Количество теплоты пропорционально току, проходящему через проводники. Единичным элементом модуля Пельтье является термопара, включающая в себя одну ветвь p-типа и одну ветвь n-типа. Ветви соединяются между собой при помощи коммутационной пластины из меди. В качестве материала ветвей традиционно используются полупроводники на основе висмута, теллура, сурьмы, селена. Модуль Пельтье представляет собой совокупность термопар, электрически соединенных, как правило, последовательно.

Термопары помещаются между двух плоских керамических пластин на основе оксида алюминия. Количество термопар может изменяться в широких пределах (от нескольких до сотен пар), что позволяет создавать модули практически любой холодильной мощности — от десятых долей до сотен ватт. В результате прохождения постоянного электрического тока между сторонами модуля образуется перепад температур: одна пластина (холодная) охлаждается, а другая (горячая) нагревается. С горячей стороны необходимо обеспечить эффективный отвод тепла в окружающую среду, например, с помощью радиатора. Тогда на холодной стороне можно получить температуру, которая будет на десятки градусов ниже температуры окружающей среды (максимум 60 градусов). Степень охлаждения будет пропорциональной величине тока. При смене полярности постоянного тока горячая и холодная пластины меняются местами. Модули Пельтье применяются в компьютерной технике, радиоэлектронных устройствах, медицинском и фармацевтическом оборудовании, бытовой технике, климатическом оборудовании.

Пример маркировки модуля Пельтье:

Где:

  • 1-ТЕС1 – термоэлемент
  • 2-Количество термоэлементов
  • 3-Максимальный ток/10, A
  • 4-Размер (40=40*40 мм)

Подобрать термоэлектрические модули и элементы Пельтье в нашем каталоге

Теплота и электрический ток

Теплота и электрический ток

ТЕРМОЭЛЕКТРИЧЕСКИЙ ЭФФЕКТ

Начиная с 1794 г. Вольта многократно проводил такой опыт: он помещал лягушку, препарированную по методу Гальвани, таким образом, чтобы ее задние лапки были опущены в воду одной банки, а спинка и позвоночный столб погружались в другую банку с водой. Если цепь замыкалась железной проволокой, один конец которой опускался на несколько минут в кипящую воду, то наблюдались сильные конвульсии лягушки, продолжавшиеся до тех пор, пока конец проволоки не остывал.

Этот опыт прошел незамеченным, и о нем, вероятно, ничего не знал также Томас Зеебек (1770—1831), выступивший в 1821 г. с докладами в Берлинской Академии наук. На основе этих докладов Зеебеком впоследствии была напитана известная работа, вышедшая лишь в 1825 г. Открытое им явление теперь хорошо известно. Сам Зеебек описывает один из своих многочисленных  опытов следующим образом. Небольшой кусок висмута был припаян с обоих концов к медной спирали. Если один конец нагревался с помощью лампы, а другой оставался холодным, то магнитная стрелка, заключенная внутри спирали, поворачивалась, указывая на прохождение тока, который в холодном спае шел от меди к висмуту.

Это явление стало известно в 1823 г. благодаря Эрстеду, который и дал ему название, укоренившееся до наших дней. В том же 1823 г. Фурье и Эрстед доказали, что термоэлектрический эффект обладает свойством суперпозиции, и построили первую термоэлектрическую батарею, состоявшую из трех пластин сурьмы, чередовавшихся с тремя пластинами висмута и спаянных на концах так, что они образовывали шестиугольник. Эта батарея была значительно усовершенствована в 1829 г. Нобили, который расположил биметаллические палочки, соединив их не торцами, а плоскостями, в наклонном положении, почти вертикально, по краям цилиндрической поверхности и поместил в сосуд, залитый камедью, так что одна группа спаев была погружена в камедь, а другая выступала наружу. Дальнейшее усовершенствование было внесено через год Меллони, сконструировавшим призматическую модель, используемую и сейчас. На основе батареи Меллони и гальванометра своей конструкции Нобили построил в том же 1830 г. термомультипликатор такой чувствительности, что он реагировал на тепло человеческого тела  на расстоянии 18—20 локтей.

В 1834 г. в ходе экспериментальных исследований проводимости сурьмы и висмута Жан Шарль Пельтье (1785—1845) намеревался определить, как изменяется температура вдоль однородного или разнородного проводника, по которому проходит ток. В связи с этим Пельтье исследовал температуру в разных точках термоэлектрической цепи с помощью термопары, соединенной с гальванометром, причем обнаружил, что в местах спаев разных металлов температура резко меняется, имеются даже случаи охлаждения. Наибольшего эффекта ему удалось добиться с парой висмут — сурьма. Таким образом, электроток может вызывать и охлаждение. Беккерель, Де ла Рив и другие физики отнеслись с недоверием к опытам Пельтье, отчасти, вероятно, потому, что он был в науке, так сказать, случайным человеком — до тридцати лет Пельтье был часовщиком. Чтобы исключить всякие сомнения, Пельтье подтвердил открытое им явление непосредственно с помощью воздушного термометра. Именно этот метод и сейчас описывается в учебниках. В других своих опытах Пельтье спаивал накрест два куска металла, затем, подключив гальванометр, пропускал через два последовательных конца креста и через гальванометр термоэлектрический ток, а спустя некоторое время цепь разъединял и подсоединял тот же гальванометр, но к другим двум концам креста — и гальванометр показывал ток, вызывавшийся либо нагреванием, либо охлаждением спая креста. В каких случаях получается нагревание, а в каких охлаждение в месте спая, точно определил в 1838 г. Поггендорф и независимо от него в 1840 г. Луиджи Пачинотти (1807—1889), отец Антонио Пачинотти, изобретателя динамо-машины постоянного тока.


ЗАКОН ДЖОУЛЯ


В течение первых сорока лет после изобретения батареи предпринималось множество попыток, частью неудачных, а частью незавершенных, выяснить, какому закону подчиняется выделение тепла электрическим током. Неудачи этих попыток можно объяснить недостаточной ясностью понятий силы тока и электрического сопротивления и как следствие — отсутствием точно определенных единиц измерения. К тому же из-за незнания закона Ома исследователи подключали в цепь последовательно провода с разным сопротивлением, считая, что они тем самым изменяют только сопротивление, а не силу тока. Этим объясняется неудача некоторых исследований, таких, как исследования Уильяма Харриса (1791 — 1867), которые, как стало ясно теперь, вполне могли привести к желаемой цели.

В 1841 г. Джоуль начал экспериментальное исследование теплоты, выделяемой проводником. Ему пришла удачная мысль прокалибровать сначала свою тангенс-буссоль в цепи с вольтаметром, как это предлагал делать Фарадей. Нагревающее приспособление состояло из исследуемого проводника, обмотанного спиралью вокруг тонкой стеклянной трубки, погруженной в стеклянный резервуар с определенным количеством воды, и чувствительного термометра. В трех проводившихся опытах, в каждом из которых последовательно соединялись два сопротивления, погруженные в одинаковые калориметры, Джоуль установил, что при одной и той же силе тока количество выделяемой теплоты пропорционально сопротивлениям проводников.

Этот первый результат привел его к формулировке гипотезы о влиянии силы тока. Он выразил ее в таком не очень ясном рассуждении: «Размышляя над вышеуказанным законом, я подумал, что действие тока должно изменяться при увеличении силы электрического тока как квадрат силы тока, потому что ясно, что в таком случае сопротивление должно изменяться в двойном отношении: из-за увеличения количества проходящего электричества в данный промежуток времени, а также из-за увеличения самой его скорости».

Джоуль, вероятно, хотел сказать, что теплота, выделяемая током, вызывается ударами частиц электрического флюида о частицы проводника. Поэтому, если увеличивается сила тока, увеличивается скорость частиц электрического флюида и удары получаются более сильными, а также более частыми вследствие увеличения количества электрического флюида, проходящего за данный промежуток времени через сечение проводника.

Но как бы там ни было, Джоуль подверг свою гипотезу опытной проверке и обнаружил, что количество тепла, измеренное калориметром, в который была погружена медная спираль, столь мало отличалось от расчетного, что можно было признать закон вполне подтвержденным, по крайней мере для металлических  проводников.

Гораздо более оригинальными были опыты, проведенные Джоулем для проверки этого закона для токов в электролитах и для токов индукции. Результаты этих исследований были изложены в работе 1843 г. В этой работе устанавливается, что в любом случае, с любым проводником, при любом токе выделяемое тепло пропорционально сопротивлению проводника и квадрату силы тока.

Естественно, что многие ученые повторили опыты Джоуля, видоизменяя их, и подтвердили полученные Джоулем результаты, выведя из них первые следствия. Среди этих следствий мы упомянем лишь результат, полученный в 1844 г. в Петербурге Ленцем и независимо от него в 1845 г. профессором физики в Турине Доменико Ботто (1791—1865). Эти исследователи установили, что генератор может отдать во внешнюю цепь максимальное количество тепла, если сопротивление цепи равно внутреннему сопротивлению генератора. Именно в этой связи Ленц начал нелегкую работу по определению зависимости температуры нагрева проводника от проходящего по нему тока и от среды, в которой он находится.

Составила к.т.н. Савельева Ф.Н.


11.2 Тепло, удельная теплоемкость и теплопередача — физика

Теплообмен, удельная теплоемкость и теплоемкость

В предыдущем разделе мы узнали, что температура пропорциональна средней кинетической энергии атомов и молекул в веществе, и что средняя внутренняя кинетическая энергия вещества тем выше, чем выше температура вещества.

Если два объекта с разной температурой соприкасаются друг с другом, энергия передается от более горячего объекта (то есть объекта с более высокой температурой) к более холодному (с более низкой температурой) объекту, пока оба объекта не будут иметь одинаковую температуру. .Когда температуры равны, нетто-теплопередачи, поскольку количество тепла, передаваемого от одного объекта к другому, равно количеству возвращаемого тепла. Одним из основных эффектов теплопередачи является изменение температуры: нагревание увеличивает температуру, а охлаждение снижает ее. Эксперименты показывают, что тепло, передаваемое веществу или от него, зависит от трех факторов: изменения температуры вещества, массы вещества и определенных физических свойств, связанных с фазой вещества.

Уравнение теплопередачи Q равно

Q = mcΔT, Q = mcΔT,

11,7

, где м — масса вещества, а Δ T — изменение его температуры в единицах Цельсия или Кельвина. Обозначение c обозначает удельную теплоемкость и зависит от материала и фазы. Удельная теплоемкость — это количество тепла, необходимое для изменения температуры 1,00 кг массы на 1,00 ºC. Удельная теплоемкость c — это свойство вещества; его единица СИ — Дж / (кг К) или Дж / (кг ° C ° C).Изменение температуры (ΔTΔT) одинаково в кельвинах и градусах Цельсия (но не в градусах Фаренгейта). Удельная теплоемкость тесно связана с понятием теплоемкости. Теплоемкость — это количество тепла, необходимое для изменения температуры вещества на 1,00 ° C ° C. В форме уравнения теплоемкость C равна C = mcC = mc, где m — масса, а c — удельная теплоемкость. Обратите внимание, что теплоемкость такая же, как и удельная теплоемкость, но без какой-либо зависимости от массы.Следовательно, два объекта, состоящие из одного и того же материала, но с разной массой, будут иметь разную теплоемкость. Это связано с тем, что теплоемкость — это свойство объекта, а удельная теплоемкость — это свойство любого объекта , изготовленного из того же материала.

Значения удельной теплоемкости необходимо искать в таблицах, потому что нет простого способа их вычислить. В таблице 11.2 приведены значения удельной теплоемкости для некоторых веществ в качестве справочной информации. Из этой таблицы видно, что удельная теплоемкость воды в пять раз больше, чем у стекла, а это означает, что для повышения температуры 1 кг воды требуется в пять раз больше тепла, чем для повышения температуры 1 кг стекла тем же способом. количество градусов.

Поддержка учителей

Поддержка учителей

[BL] [OL] [AL] Объясните, что эта формула работает только тогда, когда фаза вещества не меняется. Передача тепловой энергии, тепла и фазовый переход будут рассмотрены позже в этой главе.

Предупреждение о заблуждении

Единицы измерения удельной теплоемкости — Дж / (кг ° C⋅ ° C) и Дж / (кг K). Однако градусы Цельсия и Кельвина не всегда взаимозаменяемы. В формуле для удельной теплоемкости используется разница в температуре, а не абсолютная температура.Это причина того, что градусы Цельсия могут использоваться вместо Кельвина.

Вещества Удельная теплоемкость ( c )
Твердые вещества Дж / (кг ° C⋅ ° C)
Алюминий 900
Асбест 800
Бетон, гранит (средний) 840
Медь 387
Стекло 840
Золото 129
Тело человека (среднее) 3500
Лед (средний) 2090
Чугун, сталь 452
Свинец 128
Серебро 235
Дерево 1700
Жидкости
Бензол 1740
этанол 2450
Глицерин 2410
Меркурий 139
Вода 4186
Газы (при постоянном давлении 1 атм)
Воздух (сухой) 1015
Аммиак 2190
Двуокись углерода 833
Азот 1040
Кислород 913
Пар 2020

Таблица 11.2 Удельная теплоемкость различных веществ.

Snap Lab

Изменение температуры земли и воды

Что нагревается быстрее, земля или вода? Вы ответите на этот вопрос, проведя измерения для изучения различий в удельной теплоемкости.

  • Открытое пламя — соберите все распущенные волосы и одежду перед тем, как зажечь открытое пламя. Следуйте всем инструкциям своего учителя о том, как зажечь пламя. Никогда не оставляйте открытое пламя без присмотра. Знайте расположение противопожарного оборудования в лаборатории.
  • Песок или грунт
  • Вода
  • Духовка или тепловая лампа
  • Две маленькие баночки
  • Два термометра

Инструкции

Процедура

  1. Поместите равные массы сухого песка (или почвы) и воды одинаковой температуры в две небольшие банки. (Средняя плотность почвы или песка примерно в 1,6 раза больше плотности воды, поэтому вы можете получить равные массы, используя на 50 процентов больше воды по объему.)
  2. Нагрейте оба вещества (с помощью духовки или нагревательной лампы) в течение одинакового времени.
  3. Запишите конечные температуры двух масс.
  4. Теперь доведите обе банки до одинаковой температуры, нагревая их в течение более длительного периода времени.
  5. Снимите банки с источника тепла и измеряйте их температуру каждые 5 минут в течение примерно 30 минут.

Проверка захвата

Потребовалось больше времени, чтобы нагреть воду или песок / почву до той же температуры? Какой образец остыл дольше? Что этот эксперимент говорит нам о том, как удельная теплоемкость воды по сравнению с удельной теплотой земли?

  1. Песок / почва нагревается и остывает дольше.Это говорит нам о том, что удельная теплоемкость земли больше, чем у воды.
  2. Песок / почва нагревается и остывает дольше. Это говорит нам о том, что удельная теплоемкость воды больше, чем у земли.
  3. Вода нагревается и остывает дольше. Это говорит нам о том, что удельная теплоемкость земли больше, чем у воды.
  4. Вода нагревается и остывает дольше. Это говорит нам о том, что удельная теплоемкость воды больше, чем у земли.

Проводимость, конвекция и излучение

Всякий раз, когда есть разница температур, происходит передача тепла. Передача тепла может происходить быстро, например, через сковороду, или медленно, например, через стенки изолированного холодильника.

Существует три различных метода теплопередачи: теплопроводность, конвекция и излучение. Иногда все три могут происходить одновременно. См. Рисунок 11.3.

Рис. 11.3 В камине передача тепла происходит всеми тремя способами: теплопроводностью, конвекцией и излучением.Излучение отвечает за большую часть тепла, передаваемого в комнату. Передача тепла также происходит через теплопроводность в комнату, но гораздо медленнее. Теплообмен за счет конвекции также происходит через холодный воздух, поступающий в комнату вокруг окон, и горячий воздух, покидающий комнату, поднимаясь вверх по дымоходу.

Проводимость — это передача тепла при прямом физическом контакте. Тепло, передаваемое между электрической горелкой плиты и дном сковороды, передается за счет теплопроводности. Иногда мы пытаемся контролировать теплопроводность, чтобы чувствовать себя более комфортно.Поскольку скорость теплопередачи у разных материалов разная, мы выбираем такие ткани, как толстый шерстяной свитер, которые зимой замедляют отвод тепла от нашего тела.

Когда вы идете босиком по ковру в гостиной, ваши ноги чувствуют себя относительно комфортно… пока вы не ступите на кафельный пол кухни. Поскольку ковер и кафельный пол имеют одинаковую температуру, почему один из них холоднее другого? Это объясняется разной скоростью теплопередачи: материал плитки отводит тепло от вашей кожи с большей скоростью, чем ковровое покрытие, что делает его на холоднее.

Поддержка учителей

Поддержка учителей

[BL] [OL] [AL] Спросите учащихся, какая сейчас температура в классе. Спросите их, все ли предметы в комнате имеют одинаковую температуру. Как только это будет установлено, попросите их положить руку на стол или на металлический предмет. Стало холоднее? Почему? Если их стол сделан из ламината Formica, то рука будет прохладной, потому что ламинат является хорошим проводником тепла и отводит тепло от руки, создавая ощущение «холода» из-за тепла, покидающего тело.

Некоторые материалы просто проводят тепловую энергию быстрее, чем другие. В целом металлы (например, медь, алюминий, золото и серебро) являются хорошими проводниками тепла, тогда как такие материалы, как дерево, пластик и резина, плохо проводят тепло.

На рисунке 11.4 показаны частицы (атомы или молекулы) в двух телах при разных температурах. (Средняя) кинетическая энергия частицы в горячем теле выше, чем в более холодном теле. Если две частицы сталкиваются, энергия передается от частицы с большей кинетической энергией к частице с меньшей кинетической энергией.Когда два тела находятся в контакте, происходит много столкновений частиц, что приводит к чистому потоку тепла от тела с более высокой температурой к телу с более низкой температурой. Тепловой поток зависит от разности температур ΔT = Thot-TcoldΔT = Thot-Tcold. Таким образом, вы получите более сильный ожог от кипятка, чем от горячей воды из-под крана.

Рис. 11.4 Частицы в двух телах при разных температурах имеют разные средние кинетические энергии. Столкновения, происходящие на контактной поверхности, имеют тенденцию передавать энергию из высокотемпературных областей в низкотемпературные области.На этой иллюстрации частица в области более низких температур (правая сторона) имеет низкую кинетическую энергию перед столкновением, но ее кинетическая энергия увеличивается после столкновения с контактной поверхностью. Напротив, частица в области более высоких температур (слева) имеет большую кинетическую энергию до столкновения, но ее энергия уменьшается после столкновения с контактной поверхностью.

Конвекция — это передача тепла движением жидкости. Такой тип теплопередачи происходит, например, в котле, кипящем на плите, или во время грозы, когда горячий воздух поднимается к основанию облаков.

Советы для успеха

В обиходе термин жидкость обычно означает жидкость. Например, когда вы заболели и врач говорит вам «выпить жидкости», это означает только пить больше напитков, а не вдыхать больше воздуха. Однако в физике жидкость означает жидкость или газ . Жидкости движутся иначе, чем твердый материал, и даже имеют свой собственный раздел физики, известный как гидродинамика , который изучает их движение.

При повышении температуры жидкости они расширяются и становятся менее плотными.Например, на рис. 11.4 может быть изображена стенка воздушного шара с газами внутри воздушного шара с другой температурой, чем снаружи в окружающей среде. Более горячие и, следовательно, быстро движущиеся частицы газа внутри воздушного шара ударяются о поверхность с большей силой, чем более холодный воздух снаружи, вызывая расширение воздушного шара. Это уменьшение плотности по отношению к окружающей среде создает плавучесть (тенденцию к повышению). Конвекция обусловлена ​​плавучестью — горячий воздух поднимается вверх, потому что он менее плотен, чем окружающий воздух.

Иногда мы контролируем температуру своего дома или самих себя, контролируя движение воздуха. Герметизация дверей герметичным уплотнением защищает от холодного ветра зимой. Дом на рис. 11.5 и горшок с водой на плите на рис. 11.6 являются примерами конвекции и плавучести, созданными человеком. Океанские течения и крупномасштабная атмосферная циркуляция переносят энергию из одной части земного шара в другую и являются примерами естественной конвекции.

Рисунок 11.5 Воздух, нагретый так называемой гравитационной печью, расширяется и поднимается вверх, образуя конвективную петлю, которая передает энергию другим частям комнаты. По мере того, как воздух охлаждается у потолка и внешних стен, он сжимается, в конечном итоге становясь более плотным, чем воздух в помещении, и опускается на пол. Правильно спроектированная система отопления, подобная этой, в которой используется естественная конвекция, может быть достаточно эффективной для равномерного обогрева дома.

Рис. 11.6 Конвекция играет важную роль в теплопередаче внутри этого котла с водой.Попав внутрь жидкости, теплопередача к другим частям кастрюли происходит в основном за счет конвекции. Более горячая вода расширяется, уменьшается по плотности и поднимается, передавая тепло другим областям воды, в то время как более холодная вода опускается на дно. Этот процесс повторяется до тех пор, пока в кастрюле есть вода.

Излучение — это форма передачи тепла, которая происходит при испускании или поглощении электромагнитного излучения. Электромагнитное излучение включает радиоволны, микроволны, инфракрасное излучение, видимый свет, ультрафиолетовое излучение, рентгеновские лучи и гамма-лучи, все из которых имеют разные длины волн и количество энергии (более короткие волны имеют более высокую частоту и больше энергии).

Поддержка учителей

Поддержка учителей

[BL] [OL] Электромагнитные волны также часто называют электромагнитными волнами. Мы по-разному воспринимаем электромагнитные волны разной частоты. Так же, как мы можем видеть одни частоты как видимый свет, мы воспринимаем некоторые другие как тепло.

Вы можете почувствовать теплоотдачу от огня и солнца. Точно так же вы иногда можете сказать, что духовка горячая, не касаясь ее дверцы и не заглядывая внутрь — она ​​может просто согреть вас, когда вы пройдете мимо.Другой пример — тепловое излучение человеческого тела; люди постоянно излучают инфракрасное излучение, которое не видно человеческому глазу, но ощущается как тепло.

Излучение — единственный метод передачи тепла, при котором среда не требуется, а это означает, что тепло не должно вступать в прямой контакт с какими-либо предметами или переноситься ими. Пространство между Землей и Солнцем в основном пусто, без какой-либо возможности теплопередачи за счет конвекции или теплопроводности. Вместо этого тепло передается за счет излучения, и Земля нагревается, поскольку она поглощает электромагнитное излучение, испускаемое Солнцем.

Рис. 11.7 Большая часть тепла от этого пожара передается наблюдателям через инфракрасное излучение. Видимый свет передает относительно небольшую тепловую энергию. Поскольку кожа очень чувствительна к инфракрасному излучению, вы можете почувствовать присутствие огня, даже не глядя на него. (Дэниел X. О’Нил)

Все объекты поглощают и излучают электромагнитное излучение (см. Рисунок 11.7). Скорость передачи тепла излучением в основном зависит от цвета объекта. Черный — наиболее эффективный поглотитель и радиатор, а белый — наименее эффективный.Например, люди, живущие в жарком климате, обычно избегают ношения черной одежды. Точно так же черный асфальт на стоянке будет горячее, чем прилегающие участки травы в летний день, потому что черный поглощает лучше, чем зеленый. Верно и обратное — черный цвет излучает лучше, чем зеленый. Ясной летней ночью черный асфальт будет холоднее, чем зеленый участок травы, потому что черный излучает энергию быстрее, чем зеленый. Напротив, белый цвет — плохой поглотитель и плохой радиатор. Белый объект, как зеркало, отражает почти все излучение.

Поддержка учителя

Поддержка учителя

Попросите учащихся привести примеры теплопроводности, конвекции и излучения.

Виртуальная физика

Формы и изменения энергии

В этой анимации вы исследуете теплопередачу с различными материалами. Поэкспериментируйте с нагревом и охлаждением железа, кирпича и воды. Для этого перетащите объект на пьедестал и затем удерживайте рычаг в положении «Нагреть» или «Охлаждать». Перетащите термометр рядом с каждым объектом, чтобы измерить его температуру — вы можете в реальном времени наблюдать за тем, как быстро он нагревается или охлаждается.

Теперь попробуем передать тепло между объектами. Нагрейте кирпич и поместите его в прохладную воду. Теперь снова нагрейте кирпич, но затем поместите его поверх утюга. Что ты заметил?

Выбор опции быстрой перемотки вперед позволяет ускорить передачу тепла и сэкономить время.

Проверка захвата

Сравните, насколько быстро различные материалы нагреваются или охлаждаются. Основываясь на этих результатах, какой материал, по вашему мнению, имеет наибольшую удельную теплоемкость? Почему? Какая из них имеет наименьшую удельную теплоемкость? Можете ли вы представить себе реальную ситуацию, в которой вы хотели бы использовать объект с большой удельной теплоемкостью?

  1. Вода занимает больше всего времени, а железу нужно меньше времени, чтобы нагреться и остыть.Для теплоизоляции желательны объекты с большей удельной теплоемкостью. Например, шерстяная одежда с большой удельной теплоемкостью предотвратит потерю тепла телом.
  2. Вода занимает меньше всего времени, а железу нужно больше времени, чтобы нагреться и остыть. Для изоляции желательны объекты с большей удельной теплоемкостью. Например, шерстяная одежда с большой удельной теплоемкостью предотвратит потерю тепла телом.
  3. Кирпич занимает меньше всего времени, а железу нужно больше времени, чтобы нагреться и остыть.Для изоляции желательны объекты с большей удельной теплоемкостью. Например, шерстяная одежда с большой удельной теплоемкостью предотвратит потерю тепла телом.
  4. Вода занимает меньше всего времени, а кирпичу нужно больше времени, чтобы нагреться и остыть. Для изоляции желательны объекты с большей удельной теплоемкостью. Например, шерстяная одежда с большой удельной теплоемкостью предотвратит потерю тепла телом.
Поддержка учителя
Поддержка учителя

Попросите учащихся рассмотреть различия в результатах интерактивных упражнений при использовании разных материалов.Например, спросите их, будет ли изменение температуры больше или меньше, если кирпич заменить железным блоком той же массы, что и кирпич. Попросите студентов рассмотреть одинаковые массы металлов, алюминия, золота и меди. После того, как они заявят, больше или меньше изменение температуры для каждого металла, попросите их обратиться к Таблице 11.2 и проверить, верны ли их прогнозы.

проводимости | Физика

Цели обучения

К концу этого раздела вы сможете:

  • Рассчитайте теплопроводность.
  • Наблюдать за теплопроводностью при столкновении.
  • Изучение теплопроводности обычных веществ.

Рис. 1. Изоляция используется для ограничения теплопроводности изнутри наружу (зимой) и снаружи внутрь (летом). (кредит: Джайлз Дуглас)

Вам холодно в ногах, когда вы идете босиком по ковру в гостиной в холодном доме, а затем ступаете на плиточный пол кухни. Этот результат интригует, так как ковер и кафельный пол имеют одинаковую температуру.Различные ощущения, которые вы испытываете, объясняются разной скоростью теплопередачи: потери тепла в течение одного и того же промежутка времени больше для кожи, контактирующей с плиткой, чем с ковром, поэтому перепад температуры больше на плитке.

Некоторые материалы проводят тепловую энергию быстрее, чем другие. В целом, хорошие проводники электричества (металлы, такие как медь, алюминий, золото и серебро) также являются хорошими проводниками тепла, тогда как изоляторы электричества (дерево, пластик и резина) являются плохими проводниками тепла.На рисунке 2 показаны молекулы в двух телах при разных температурах. (Средняя) кинетическая энергия молекулы в горячем теле выше, чем в более холодном теле. Если две молекулы сталкиваются, происходит передача энергии от горячей молекулы к холодной. Кумулятивный эффект от всех столкновений приводит к чистому потоку тепла от горячего тела к более холодному телу. Таким образом, тепловой поток зависит от разности температур Δ = Τ горячий T холодный .Таким образом, вы получите более сильный ожог от кипятка, чем от горячей воды из-под крана. И наоборот, если температуры одинаковы, чистая скорость теплопередачи падает до нуля и достигается равновесие. Благодаря тому, что количество столкновений увеличивается с увеличением площади, теплопроводность зависит от площади поперечного сечения. Если прикоснуться ладонью к холодной стене, рука остынет быстрее, чем при прикосновении к ней кончиком пальца.

Рис. 2. Молекулы в двух телах при разных температурах имеют разные средние кинетические энергии.Столкновения, происходящие на контактной поверхности, имеют тенденцию передавать энергию из высокотемпературных областей в низкотемпературные области. На этом рисунке молекула в области более низких температур (правая сторона) имеет низкую энергию перед столкновением, но ее энергия увеличивается после столкновения с контактной поверхностью. Напротив, молекула в области более высоких температур (слева) имеет высокую энергию до столкновения, но ее энергия уменьшается после столкновения с контактной поверхностью.

Третий фактор в механизме теплопроводности — это толщина материала, через который передается тепло.На рисунке ниже показана плита из материала с разными температурами с обеих сторон. Предположим, что T 2 больше, чем T 1 , так что тепло передается слева направо. Передача тепла с левой стороны на правую осуществляется серией столкновений молекул. Чем толще материал, тем больше времени требуется для передачи того же количества тепла. Эта модель объясняет, почему толстая одежда зимой теплее, чем тонкая, и почему арктические млекопитающие защищаются толстым салом.

Рис. 3. Теплопроводность происходит через любой материал, представленный здесь прямоугольной полосой, будь то оконное стекло или моржовый жир. Температура материала составляет T 2 слева и T 1 справа, где T 2 больше, чем T 1 . Скорость теплопередачи прямо пропорциональна площади поверхности A, разности температур T 2 T 1 и проводимости вещества k .Скорость теплопередачи обратно пропорциональна толщине d .

Наконец, скорость теплопередачи зависит от свойств материала, описываемых коэффициентом теплопроводности. Все четыре фактора включены в простое уравнение, выведенное из экспериментов и подтвержденное экспериментами. Скорость кондуктивной теплопередачи через пластину материала, такую ​​как показанная на рисунке 3, равна

.

[латекс] \ displaystyle \ frac {Q} {t} = \ frac {kA \ left (T_2-T_1 \ right)} {d} \\ [/ latex],

, где [latex] \ frac {Q} {t} \\ [/ latex] — скорость теплопередачи в ваттах или килокалориях в секунду, k — теплопроводность материала, A и d — это его площадь поверхности и толщина, как показано на Рисунке 3, а ( T 2 T 1 ) — разность температур на плите.В таблице 1 приведены типичные значения теплопроводности.

Пример 1. Расчет теплопроводности: скорость теплопроводности через ледяной ящик

Ледяной ящик из пенополистирола имеет общую площадь 0,950 м 2 и среднюю толщину стенок 2,50 см. В коробке есть лед, вода и напитки в банках с температурой 0 ° C. Внутренняя часть ящика охлаждается за счет таяния льда. Сколько льда тает за сутки, если хранить ледяной ящик в багажнике автомобиля при температуре 35,0ºC?

Стратегия

Этот вопрос включает как тепло для фазового перехода (таяние льда), так и передачу тепла за счет теплопроводности.{\ circ} \ text {C}; \\ t & = & 1 \ text {day} = 24 \ text {hours} = 86 400 \ text {s}. \ end {array} \\ [/ latex]

Определите неизвестные. Нам нужно найти массу льда м . Нам также нужно будет вычислить чистое тепло, передаваемое для плавления льда, Q . Определите, какие уравнения использовать. Скорость теплопередачи за счет теплопроводности определяется как

.

[латекс] \ displaystyle \ frac {Q} {t} = \ frac {kA \ left (T_2-T_1 \ right)} {d} \\ [/ latex]

Тепло используется для плавления льда: Q мл f .{\ circ} \ text {C} \ right)} {0,0250 \ text {m}} = 13,3 \ text {J / s} \\ [/ latex]

Умножьте скорость теплопередачи на время (1 день = 86 400 с): Q = [латекс] \ left (\ frac {Q} {t} \ right) t \\ [/ latex] = ( 13,3 Дж / с) (86400 с) = 1,15 × 10 6 Дж

Установите равным теплу, передаваемому для таяния льда: Q = мл f . Решим относительно массы м :

[латекс] \ displaystyle {m} = \ frac {Q} {L _ {\ text {f}}} = \ frac {1.3 \ text {Дж / кг}} = 3,44 \ text {кг} \\ [/ latex]

Обсуждение

Результат 3,44 кг, или около 7,6 фунта, кажется вполне правильным, если судить по опыту. Вы можете рассчитывать на использование мешка льда весом около 4 кг (7–10 фунтов) в день. Если вы добавляете горячую пищу или напитки, потребуется немного льда.

Проверка проводимости в таблице 1 показывает, что пенополистирол — очень плохой проводник и, следовательно, хороший изолятор. Среди других хороших изоляторов — стекловолокно, шерсть и перья из гусиного пуха. Как и пенополистирол, все они включают в себя множество маленьких карманов с воздухом, благодаря низкой теплопроводности воздуха.

Таблица 1. Теплопроводность обычных веществ
Вещество Теплопроводность k (Дж / с⋅м⋅ºC)
Серебро 420
Медь 390
Золото 318
Алюминий 220
Стальной чугун 80
Сталь (нержавеющая) 14
Лед 2.2
Стекло (среднее) 0,84
Бетонный кирпич 0,84
Вода 0,6
Жировая ткань (без крови) 0,2
Асбест 0,16
Гипсокартон 0,16
Дерево 0,08–0,16
Снег (сухой) 0,10
Пробка 0.042
Стекловата 0,042
Шерсть 0,04
Пуховые перья 0,025
Воздух 0,023
Пенополистирол 0,010

Рис. 4. Стекловолокно используется для изоляции стен и потолков, чтобы предотвратить теплопередачу между внутренней частью здания и внешней средой.

Комбинацией материала и толщины часто манипулируют для создания хороших изоляторов — чем меньше проводимость k и чем больше толщина d , тем лучше.Соотношение [латекс] \ frac {d} {k} \\ [/ latex], таким образом, будет большим для хорошего изолятора. Соотношение [латекс] \ frac {d} {k} \\ [/ latex] называется коэффициентом R . Скорость кондуктивной теплопередачи обратно пропорциональна R . Чем больше значение R , тем лучше изоляция. Коэффициент R чаще всего указывается для бытовой теплоизоляции, холодильников и т.п. — к сожалению, он все еще выражается в неметрических единицах фут. тепловая единица [BTU] — это количество энергии, необходимое для изменения температуры на 1.0 фунтов воды при температуре 1,0 ° F). Пара типичных значений: коэффициент R , равный 11, для стекловолоконных войлоков (кусков) изоляции толщиной 3,5 дюйма и коэффициент R , равный 19, для стекловолоконных войлоков толщиной 6,5 дюймов. Стены обычно утепляются 3,5-дюймовыми ватными покрытиями, а потолки — 6,5-дюймовыми. В холодном климате для потолков и стен можно использовать более толстый войлок.

Обратите внимание, что в таблице 1 лучшие теплопроводники — серебро, медь, золото и алюминий — также являются лучшими электрическими проводниками, что опять же связано с плотностью свободных электронов в них.Кухонная утварь обычно изготавливается из хороших проводников.

Пример 2. Расчет разницы температур, поддерживаемой теплопередачей: теплопроводность через алюминиевую сковороду

Вода кипит в алюминиевой кастрюле, поставленной на электрический элемент на плите. Дно кастрюли имеет толщину 0,800 см и диаметр 14,0 см. Кипящая вода испаряется со скоростью 1,00 г / с. Какая разница температур на дне сковороды?

Стратегия

Проводимость через алюминий является здесь основным методом теплопередачи, поэтому мы используем уравнение для скорости теплопередачи и решаем разницу температур .

[латекс] \ displaystyle {T} _2-T_1 = \ frac {Q} {t} \ left (\ frac {d} {kA} \ right) \\ [/ latex]

Решение

Определите известные значения и преобразуйте их в единицы СИ Толщина поддона, d = 0,900 см = 8,0 × 10 −3 м площадь поддона, A = π (0,14 / 2) 2 м 2 = 1,54 × 10 −2 м 2 , а теплопроводность k = 220 Дж / с ⋅ м ⋅ ° C.

Рассчитайте необходимую теплоту испарения 1 г воды: Q = мл v = (1.{\ circ} \ text {C} \\ [/ latex]

Обсуждение

Значение теплопередачи [латекс] \ frac {Q} {t} \ [/ latex] = 2,26 кВт или 2256 Дж / с типично для электрической плиты. Это значение дает очень небольшую разницу температур между плитой и сковородой. Учтите, что конфорка плиты раскалена докрасна, а температура внутри сковороды почти 100ºC из-за контакта с кипящей водой. Этот контакт эффективно охлаждает дно сковороды, несмотря на его близость к очень горячей конфорке плиты.Алюминий является настолько хорошим проводником, что достаточно лишь этой небольшой разницы температур для передачи тепла в сковороду 2,26 кВт.

Проводимость возникает из-за беспорядочного движения атомов и молекул. По сути, это неэффективный механизм переноса тепла на макроскопические расстояния и короткие временные расстояния. Возьмем, к примеру, температуру на Земле, которая была бы невыносимо низкой ночью и чрезвычайно высокой днем, если бы перенос тепла в атмосфере происходил только за счет теплопроводности.В другом примере автомобильные двигатели будут перегреваться, если не будет более эффективного способа отвода избыточного тепла от поршней.

Проверьте свое понимание

Как изменяется скорость теплопередачи за счет теплопроводности при удвоении всех пространственных размеров?

Решение

Поскольку площадь является произведением двух пространственных измерений, она увеличивается в четыре раза, когда каждое измерение удваивается ( A final = (2 d ) 2 = 4 d 2 = 4 А начальный ).А расстояние просто удваивается. Поскольку разница температур и коэффициент теплопроводности не зависят от пространственных размеров, скорость передачи тепла за счет теплопроводности увеличивается в четыре раза, деленные на два или два:

[латекс] \ left (\ frac {Q} {t} \ right) _ {\ text {final}} = \ frac {kA _ {\ text {final}} \ left (T_2-T_1 \ right)} {d_ {\ text {final}}} = \ frac {k \ left (4A _ {\ text {initial}} \ right) \ left (T_2-T_1 \ right)} {2d _ {\ text {initial}}} = 2 \ frac {kA _ {\ text {initial}} \ left (T_2-T_1 \ right)} {d _ {\ text {initial}}} = 2 \ left (\ frac {Q} {t} \ right) _ {\ text {initial}} \\ [/ latex]

Сводка раздела

  • Теплопроводность — это передача тепла между двумя объектами, находящимися в непосредственном контакте друг с другом.
  • Скорость теплопередачи [латекс] \ frac {Q} {t} \\ [/ latex] (энергия в единицу времени) пропорциональна разнице температур T 2 T 1 и площадь контакта A и обратно пропорциональна расстоянию d между объектами: [latex] \ frac {Q} {t} = \ frac {\ text {kA} \ left ({T} _ {2} — {T} _ {1} \ right)} {d} \\ [/ latex].

Концептуальные вопросы

  1. Некоторые электроплиты имеют плоскую керамическую поверхность со скрытыми нагревательными элементами.Кастрюля, поставленная над нагревательным элементом, будет нагрета, при этом безопасно прикасаться к поверхности всего в нескольких сантиметрах от нее. Почему керамика с проводимостью меньше, чем у металла, но больше, чем у хорошего изолятора, является идеальным выбором для плиты?
  2. Свободная белая одежда, закрывающая большую часть тела, идеальна для обитателей пустыни как на жарком солнце, так и в холодные вечера. Объясните, чем выгодна такая одежда и днем, и ночью.

Рисунок 5.Джеллабию носят многие мужчины в Египте. (кредит: Зерида)

Задачи и упражнения

  1. (a) Рассчитайте коэффициент теплопроводности через стены дома толщиной 13,0 см, у которых средняя теплопроводность в два раза выше, чем у стекловаты. Предположим, что нет ни окон, ни дверей. Площадь стен составляет 120 м 2 2 , их внутренняя поверхность имеет температуру 18,0ºC, а их внешнюю поверхность — 5,00ºC. (b) Сколько комнатных обогревателей мощностью 1 кВт потребуется для уравновешивания теплопередачи за счет теплопроводности?
  2. Скорость передачи тепла из окна в зимний день достаточно высока, чтобы охладить воздух рядом с ним.Чтобы увидеть, насколько быстро окна передают тепло за счет теплопроводности, рассчитайте коэффициент теплопроводности в ваттах через окно размером 3,00 м 2 толщиной 0,635 см (1/4 дюйма), если температура внутренней и внешней поверхностей составляет 5,00 ºC и −10,0ºC соответственно. Такая высокая скорость не будет поддерживаться — внутренняя поверхность остынет и даже может образоваться иней.
  3. Рассчитайте скорость отвода тепла от тела человека, предполагая, что внутренняя температура ядра составляет 37,0 ° C, а температура кожи равна 34.0ºC, толщина тканей в среднем составляет 1,00 см, а площадь поверхности составляет 1,40 м 2 .
  4. Предположим, вы стоите одной ногой на керамическом полу и одной ногой на шерстяном ковре, соприкасаясь каждой ногой на площади 80,0 см. 2 . И керамика, и ковер имеют толщину 2,00 см и температуру на нижней стороне 10,0 ° C. С какой скоростью должна происходить теплопередача от каждой ступни, чтобы верхняя часть керамики и ковра поддерживала температуру 33,0 ° C?
  5. Человек потребляет 3000 ккал пищи за один день, превращая большую ее часть для поддержания температуры тела.Если он теряет половину этой энергии из-за испарения воды (при дыхании и потоотделении), сколько килограммов воды испаряется?
  6. (a) Огнеходящий бежит по раскаленному углю, не получив ожогов. Рассчитайте теплопроводность, передаваемую подошве одной ступни огнехожника, учитывая, что нижняя часть ступни представляет собой мозоль толщиной 3,00 мм с проводимостью на нижнем пределе диапазона для древесины, а ее плотность составляет 300 кг / м 2. 3 . Площадь контакта 25,0 см 2 , температура углей 700ºC, время контакта 1.00 с. (b) Какое повышение температуры происходит в 25,0 см 3 пораженной ткани? (c) Как вы думаете, какое влияние это окажет на ткань, учитывая, что каллус состоит из мертвых клеток?
  7. (а) Какова скорость теплопроводности через мех толщиной 3 см у крупного животного с площадью поверхности 1,40 м 2 ? Предположим, что температура кожи животного 32,0ºC, температура воздуха –5,00ºC и мех имеет такую ​​же теплопроводность, как воздух.(б) Какой прием пищи потребуется животному в течение одного дня, чтобы восполнить эту теплопередачу?
  8. Морж передает энергию посредством проводимости через свой жир с мощностью 150 Вт при погружении в воду с температурой –1,00 ° C. Внутренняя температура моржа составляет 37,0ºC, а его площадь поверхности составляет 2,00 м 2 . Какова средняя толщина его подкожного жира, который имеет проводимость жировых тканей без крови?

    Рис. 6. Морж на льду. (Источник: капитан Бадд Кристман, Корпус NOAA)

  9. Сравните коэффициент теплопроводности через 13.Стена толщиной 0 см, имеющая площадь 10,0 м 2 и удвоенную теплопроводность, чем у стекловаты, со скоростью теплопроводности через окно толщиной 0,750 см и площадью 2,00 м 2 , предполагая одинаковую разницу температур между ними.
  10. Предположим, что человек покрыт с головы до ног шерстяной одеждой средней толщины 2,00 см и передает энергию путем теплопроводности через одежду со скоростью 50,0 Вт. Какова разница температур в одежде, если площадь поверхности равна 1.40 м 2 ?
  11. Некоторые поверхности плит сделаны из гладкой керамики для облегчения очистки. Если керамика имеет толщину 0,600 см и теплопроводность происходит через ту же площадь и с той же скоростью, что и в примере 2, какова разница температур в ней? Керамика имеет такую ​​же теплопроводность, как стекло и кирпич.
  12. Один из простых способов сократить расходы на отопление (и охлаждение) — это добавить дополнительную изоляцию на чердаке дома. Предположим, что в доме уже есть 15 см стекловолоконной изоляции на чердаке и на всех внешних поверхностях.Если добавить на чердак еще 8,0 см стеклопластика, то на какой процент упадет стоимость отопления дома? Возьмем одноэтажный дом размером 10 м на 15 м на 3,0 м. Не обращайте внимания на проникновение воздуха и потерю тепла через окна и двери.
  13. (a) Рассчитайте коэффициент теплопроводности через окно с двойным остеклением, которое имеет площадь 1,50 м 2 и состоит из двух стекол толщиной 0,800 см, разделенных воздушным зазором 1,00 см. Температура внутренней поверхности 15.0ºC, а снаружи −10,0ºC. (Подсказка: на двух стеклянных панелях наблюдаются одинаковые перепады температуры. Сначала найдите их, а затем перепад температуры в воздушном зазоре. Эта проблема игнорирует повышенную теплопередачу в воздушном зазоре из-за конвекции.) (B) Рассчитайте скорость теплопроводность через окно толщиной 1,60 см той же площади и с такими же температурами. Сравните свой ответ с ответом на часть (а).
  14. Многие решения принимаются на основе периода окупаемости: времени, которое потребуется за счет экономии, чтобы равняться капитальным затратам на инвестиции.Приемлемые сроки окупаемости зависят от бизнеса или философии. (Для некоторых отраслей период окупаемости составляет всего два года.) Предположим, вы хотите установить дополнительную изоляцию, о которой идет речь в вопросе 12. Если стоимость энергии составляет 1 доллар США за миллион джоулей, а стоимость изоляции составляет 4 доллара США за квадратный метр, тогда рассчитайте простой срок окупаемости. . Возьмем среднее значение Δ T для 120-дневного отопительного сезона равным 15,0 ° C.
  15. Для человеческого тела, какова скорость теплопередачи через ткани тела при следующих условиях: толщина ткани 3.00 см, изменение температуры 2,00ºC, а площадь кожи 1,50 м 2 . Как это соотносится со средней скоростью передачи тепла телу в результате потребления энергии около 2400 ккал в день? (Никакие упражнения не включены.)

Глоссарий

R-фактор: отношение толщины материала к проводимости

скорость кондуктивной теплопередачи: скорость теплопередачи от одного материала к другому

теплопроводность: свойство способности материала проводить тепло

Избранные решения проблем и упражнения

1.(а) 1.01 × 10 3 Вт; (б) Один

3. 84.0 Вт

5. 2,59 кг

7. (а) 39,7 Вт; (б) 820 ккал

9. 35 к 1, окно к стене

11. 1,05 × 10 3 К

13. (а) 83 Вт; (b) в 24 раза больше, чем у окна с двойным остеклением.

15. 20,0 Вт, 17,2% от 2400 ккал в день


тепла | Определение и факты

Полная статья

Изучите теплопередачу и узнайте разницу между теплом и температурой

Узнайте о теплопередаче и взаимосвязи между теплом и температурой.

Encyclopædia Britannica, Inc. Посмотрите все видео по этой статье

тепло , энергия, которая передается от одного тела к другому в результате разницы температур. Если два тела с разной температурой сближаются, энергия передается, т. Е. Тепловые потоки, от более горячего тела к более холодному. Эффект этой передачи энергии обычно, но не всегда, заключается в повышении температуры более холодного тела и понижении температуры более горячего тела.Вещество может поглощать тепло без повышения температуры за счет перехода от одного физического состояния (или фазы) к другому, например, от твердого к жидкому (плавление), от твердого к парообразному (сублимация), от жидкости к пару. (кипение) или из одной твердой формы в другую (обычно это называется кристаллическим переходом). Важное различие между теплом и температурой (тепло является формой энергии, а температура — мерой количества этой энергии, присутствующей в теле) было прояснено в 18-19 веках.

Тепло как форма энергии

Поскольку все многочисленные формы энергии, включая тепло, могут быть преобразованы в работу, количество энергии выражается в единицах работы, таких как джоули, фут-фунты, киловатт-часы, или калорий. Существуют точные отношения между количеством тепла, добавляемого к телу или отводимого от него, и величиной воздействия на состояние тела. Две наиболее часто используемые единицы тепла — это калория и британская тепловая единица (БТЕ). Калория (или грамм-калория) — это количество энергии, необходимое для повышения температуры одного грамма воды с 14.От 5 до 15,5 ° С; BTU — это количество энергии, необходимое для повышения температуры одного фунта воды с 63 до 64 ° F. Одна БТЕ составляет примерно 252 калории. Оба определения указывают, что изменения температуры должны измеряться при постоянном давлении в одну атмосферу, поскольку количество задействованной энергии частично зависит от давления. Калория, используемая для измерения содержания энергии в продуктах, — это большая калория, или килограмм-калория, равная 1000 грамм-калориям.

Британская викторина

Викторина «Все о физике»

Кто был первым ученым, проведшим эксперимент по управляемой цепной ядерной реакции? Какая единица измерения для циклов в секунду? Проверьте свою физическую хватку с помощью этой викторины.

В общем, количество энергии, необходимое для подъема единицы массы вещества через заданный температурный интервал, называется теплоемкостью или удельной теплоемкостью этого вещества. Количество энергии, необходимое для повышения температуры тела на один градус, варьируется в зависимости от наложенных ограничений. Если тепло добавляется к газу, находящемуся в постоянном объеме, количество тепла, необходимое для повышения температуры на один градус, меньше, чем если бы тепло добавлялось к тому же газу, свободному для расширения (как в цилиндре, оснащенном подвижным поршнем). ) и так работаю.В первом случае вся энергия идет на повышение температуры газа, но во втором случае энергия не только способствует повышению температуры газа, но и обеспечивает энергию, необходимую для работы, совершаемой газом над газом. поршень. Следовательно, теплоемкость вещества зависит от этих условий. Чаще всего определяют удельную теплоемкость при постоянном объеме и удельную теплоемкость при постоянном давлении. Французские ученые Пьер-Луи Дюлон и Алексис-Тереза ​​Пети в 1819 году показали, что теплоемкость многих твердых элементов тесно связана с их атомным весом.Так называемый закон Дюлонга и Пти был полезен для определения атомного веса некоторых металлических элементов, но из него есть много исключений; Позднее было обнаружено, что отклонения можно объяснить на основе квантовой механики.

Неверно говорить о тепле в теле, потому что тепло ограничивается передачей энергии. Энергия, хранящаяся в теле, — это не тепло (и не работа, так как работа — это еще и энергия в пути). Однако принято говорить о явном и скрытом тепле.Скрытая теплота, также называемая теплотой испарения, — это количество энергии, необходимое для превращения жидкости в пар при постоянной температуре и давлении. Энергия, необходимая для плавления твердого тела в жидкость, называется теплотой плавления, а теплота сублимации — это энергия, необходимая для преобразования твердого тела непосредственно в пар, эти изменения также происходят в условиях постоянной температуры и давления.

Получите подписку Britannica Premium и получите доступ к эксклюзивному контенту. Подпишитесь сейчас

Воздух представляет собой смесь газов и водяного пара, и вода, присутствующая в воздухе, может изменять фазу; я.е., он может стать жидким (дождь) или твердым (снег). Чтобы различать энергию, связанную с фазовым переходом (скрытое тепло), и энергию, необходимую для изменения температуры, было введено понятие явного тепла. В смеси водяного пара и воздуха явное тепло — это энергия, необходимая для создания определенного изменения температуры, за исключением энергии, необходимой для фазового перехода.

Логический класс | Дом

ТОК ЭЛЕКТРИЧЕСТВА

Введение:

Когда электрический ток проходит через проводник, через некоторое время проводник нагревается и выделяет тепло.Это происходит из-за преобразования некоторой электрической энергии, проходящей через проводник, в тепловую. Этот эффект электрического тока называется тепловым эффектом тока.

Когда ток течет по проводнику, в проводнике генерируется тепловая энергия. Нагревательный эффект электрического тока зависит от трех факторов:

  • Сопротивление R проводника. Чем выше сопротивление, тем больше тепла.
  • Время t, в течение которого течет ток.Чем больше время, тем больше выделяется тепла
  • Величина тока I. Чем выше сила тока, тем больше выделяется тепла.

Следовательно, эффект нагрева, создаваемый электрическим током I через проводник сопротивления R в течение некоторого времени, t определяется как H = I 2 Rt. Это уравнение называется уравнением Джоуля электрического нагрева.

Электроэнергия и мощность

Работа, совершаемая при проталкивании заряда по электрической цепи, определяется выражением w.d =

VIt

Так что мощность, P = w.d / t = VI

Электроэнергия, потребляемая электроприбором, определяется как P = VI = I 2 R = V 2 / R

Применения теплового эффекта электрического тока

Большинство бытовых электроприборов таким образом преобразуют электрическую энергию в тепло. К ним относятся лампы накаливания, электрический нагреватель, электрический утюг, электрический чайник и т. Д.

В осветительных приборах

  1. Лампы накаливания — изготовлены из вольфрамовой проволоки, заключенной в стеклянную колбу, из которой удален воздух.Это потому, что воздух окисляет нить. Нить нагревается до высокой температуры и становится раскаленной добела. Вольфрам используется из-за его высокой температуры плавления; 3400 0 Колба заполнена неактивным газом, например. аргон или азот при низком давлении, что снижает испарение вольфрамовой проволоки. Однако одним из недостатков инертного газа является то, что он вызывает конвекционные токи, которые охлаждают нить накала. Эта проблема сводится к минимуму за счет наматывания проволоки таким образом, чтобы она занимала меньшую площадь, что снижает потери тепла за счет конвекции.
  2. Люминесцентные лампы — эти лампы более эффективны по сравнению с лампами накаливания и служат намного дольше. У них есть пары ртути в стеклянной трубке, которая при включении испускает ультрафиолетовое излучение. Это излучение заставляет порошок в трубке светиться (флуоресцировать), то есть испускать видимый свет. Из разных порошков получаются разные цвета. Обратите внимание, что люминесцентные лампы дороги в установке, но их эксплуатационные расходы намного меньше.

В электрическом нагревателе

  1. Электрические плиты — электрические плиты раскалены докрасна, и произведенная тепловая энергия поглощается кастрюлей посредством теплопроводности.
  2. Электрические обогреватели — лучистые обогреватели становятся красными при температуре около 900 0 ° C, а испускаемое излучение направляется в комнату с помощью полированных отражателей.
  3. Электрические чайники — нагревательный элемент размещается на дне чайника так, чтобы нагреваемая жидкость покрывала его. Затем тепло поглощается водой и распределяется по всей жидкости за счет конвекции.

Электрические утюги — когда через нагревательный элемент протекает ток, выделяемая тепловая энергия передается на основание из тяжелого металла, повышая его температуру.Затем эта энергия используется для прессования одежды. Температуру утюга можно контролировать с помощью термостата (биметаллической планки).

Когда между концами резистора приложена разность потенциалов V, электрический

Поле

будет действовать на свободные электроны. Работа электрического поля на бесплатном

электронов за время t равно W = Vq

Вт = V это W = i2 Rt

ii) Эта работа, совершаемая полем, преобразуется в тепловую энергию резистора через столкновения с решеткой.{2}} \)

б) Тепло, выделяемое в данном резисторе заданным током, пропорционально времени, в течение которого в нем существует ток, т. Е. \ (H \ propto t \)

c) Тепло, выделяемое в резисторе заданным током в заданное время, пропорционально его сопротивлению.

v) Электрический нагреватель, электрический утюг, электрическая лампочка, электрическая плита — вот некоторые из инструментов, которые воздействуют на тепловой эффект Джоуля и преобразуют электрическую энергию в тепловую.Эффект Джоуля необратим .

Джоулев нагрев , также известный как омический нагрев и резистивный нагрев , представляет собой процесс, при котором прохождение электрического тока через проводник производит тепло. Первый закон Джоуля , также известный как закон Джоуля-Ленца , [1] , утверждает, что мощность нагрева, генерируемая электрическим проводником, пропорциональна произведению его сопротивления и квадрата тока: Джоулевое нагревание влияет на весь электрический проводник, в отличие от эффекта Пельтье, который передает тепло от одного электрического спая к другому.

Постоянный ток :

Самая общая и основная формула для джоулева нагрева:

{\ displaystyle P = (V_ {A} -V_ {B}) I}, где

  • P — мощность (энергия в единицу времени), преобразованная из электрической энергии в тепловую,
  • I — ток, проходящий через резистор или другой элемент,
  • {\ displaystyle V_ {A} -V_ {B}} — падение напряжения на элементе.{2} / R} где R — сопротивление.

    Переменный ток :

    Основная статья: Питание переменного тока

    Когда ток меняется, как в цепях переменного тока,

    \ (P \ влево (т \ вправо) = U \ влево (т \ вправо) I \ влево (т \ вправо) \)

    , где t — время, а P — мгновенная мощность, преобразуемая из электрической энергии в тепло.

    Намного чаще средняя мощность представляет больший интерес, чем мгновенная мощность:

    \ ({{P} _ {avg}} = {{U} _ {rms}} {{I} _ {rms}} = I_ {rms} ^ {2} R = U_ {rms} ^ {2} / R \)

    {\ displaystyle P_ {avg} = U _ {\ text {rms}} I _ {\ text {rms}} = I _ {\ text {rms}} ^ {2} R = U _ {\ text {rms}} ^ { 2} / R}, где «avg» обозначает среднее значение за один или несколько циклов, а «rms» обозначает среднеквадратичное значение.{*}} \ right) \)

    , где \ (\ varnothing \) — разность фаз между током и напряжением, {\ displaystyle \ operatorname {Re}} Re означает действительную часть, Z — комплексное сопротивление, а Y * — комплексное сопряжение проводимости. (равно 1/ Z * ).

    Химическое воздействие электрического тока :

    Прохождение электрического тока через проводящую жидкость вызывает химические реакции.Возникающие в результате эффекты получили название химического

    .

    воздействия токов. Процесс нанесения слоя любого желаемого металла на другой материал с помощью электричества называется гальваникой.

    1. Найдите энергию, рассеиваемую за 5 минут электрической лампочкой с нитью накала 500 Ом, подключенной к источнику питания 240 В.

    Решение:

    E = Pt = V2 / R * t = (240 2 * 5 * 60) / 500 = 34,560 Дж.

    2. Электрическая лампочка имеет маркировку 100 Вт, 240 В. Вычислить:
    и) Ток через нить накала
    ii) Сопротивление нити накала лампы
    .

    Решение:

    P = VI I = P / V = ​​100/240 = 0,4167A
    По закону Ома V = IR R = V / I = 240 / 0,4167 = 575,95 Ом.

    1. Для нагрева воды используется погружной нагреватель мощностью 2,5 кВт. Вычислить:

    i) Рабочее напряжение нагревателя, если его сопротивление составляет 24 Ом

    ii) Преобразование электрической энергии в тепловую за 2 часа.

    Решение

    P = VI = I 2 R

    I = (2500/24) 1/2 = 10,2062A

    В = ИК = 10,2062 * 24 = 244,9488 В

    E = VIt = Pt = 2500 * 2 * 60 * 60 = 1,8 * 10 7 Дж

    ИЛИ E = VIt = 244,9488 * 10,2062 * 2 * 60 * 60 = 1,8 * 10 7 Дж.

    1. Найдите напряжение на клеммах E 1 и E 2 , как показано на рисунке.

    Решение:

    Ток на рисунке

    \ (I = \ frac {9-3} {10} = 0,6A \)

    \ ({{V} _ {1}} = {{E} _ {1}} + I {{r} _ {1}} = 3 + 0,6 \ times 1 = 3,6 В; {{V} _ {2}} = {{E} _ {2}} — Я {{r} _ {2}} = 9-0,6 \ умножить на 2 = 7,8 В \)

    2. Рассчитайте установившийся ток в показанном резисторе 2 Вт .Внутреннее Сопротивление батареи незначительно, а емкость конденсатора 0,2 м F.

    Решение:

    Сопротивление параллельной комбинации резисторов 2 Вт и 3 Вт равно

    .

    \ (\ frac {1} {R} = \ frac {1} {2} + \ frac {1} {3} = \ frac {5} {6} \ Rightarrow \, \, \, \, \, R = 1.2 \, \, \ Omega \)

    Это сопротивление последовательно с 2,8 W , что дает общее эффективное сопротивление

    = 1,2 + 2,8 W = 4 W . {2}} {Power} \, \, = \, \, \ frac {100 \, x \, 100} {1000} \, \, \, = \, \, 100 \, \ Omega \)

    И как рассеивается 62.{2}} {{{R} _ {H}}} \, \, = \, \, 62.5 \)

    В В = 25 В.

    .

    Тепло человеческого тела как источник термоэлектрической энергии

    Тепло человеческого тела как источник термоэлектрической энергии

    Мэтью Стивенс


    27 ноября 2016

    Представлено как курсовая работа для Ph340, Стэнфордский университет, осень 2016 г.

    Введение

    Рис.1: Принципиальная схема термоэмиссионный генератор электричества, который может быть использован сделать силу из человеческого тела. (Источник: Викимедиа Commons)

    Идея преобразования тепла человеческого тела в форму полезная энергия была целью ученых в течение многих лет. Отдыхающий человек самец выделяет примерно 100-120 Вт энергии. Очень небольшая часть это может быть использовано термоэлектрическим устройством для питания носимых устройств.Например, часы Seiko Thermic Watch смогли запечатлеть непрерывно заряжается всего на один микроватт, поэтому, безусловно, есть жизнеспособный рынок, который еще не расцвел. Известно, что 80% сила обычного человеческого тела выделяется в виде тепла, современные технологии может производить только несколько милливатт энергии. Этого недостаточно чтобы зарядить iPhone при входной мощности примерно 5 Вт, и к сожалению, такая эффективность никогда не будет возможна.

    Тепло человеческого тела

    Потому что мы должны помнить, что энергия не может быть сохранены абсолютно, мы должны смотреть на происхождение этой энергии в средства питания и, в частности, калории, которые потребляет человеческий организм как источник этой тепловой энергии через обмен веществ.Единая «калорийность» пищи содержит 4,184 × 10 3 джоулей. Если средний человек потребляет 1500 калорий каждый день, что составляет 6,27 × 10 6 джоулей в день. Условно говоря, речь идет о количество энергии, необходимое для запуска автомобиля в течение 15 минут. На глобальном по шкале, это соответствует примерно 3,14 × 10 19 Дж на год. [1] Имея в виду такое количество потребляемой энергии, нам также необходимо понять вовлеченную энтропию Второй закон термодинамики устанавливает что ни одна система не является полностью эффективной, и некоторая часть этой энергии должна быть выделяется как отходящее тепло или тепло тела, как мы его знаем.

    Технологии

    Текущее термоэлектрическое преобразование энергии завершено в основном за счет использования эффекта Зеебека. На рис. 1 представлена ​​простая диаграмма, отображающая функциональный механизм. Термоэлектрические устройства генерировать энергию, когда есть разница между нагретой поверхностью (при контакте с телом) и окружающей средой. Это можно записать в форма dT / T, где dT — разница температур тела и окружающей среды, где T — температура окружающей среды в Кельвин.Если человеческое тело имеет температуру 310 ° К (98,6 ° F), и окружающая среда имеет температуру 297 ° K (75 ° F), мы получаем Значение dT составляет 13 ° K по сравнению со значением T, равным 310 ° K. Это равно значению 4,2% в доступном дифференциале. На самом деле это значение будет намного ниже. учитывая невозможность использовать все потерянное тепло, так как это было бы фатальным для любой живой человек. [2] Говоря качественно, больше электричества было бы генерируется устройством, которое пользователь носит в более холодной среде. В функции, электроны перемещаются из областей с высокой температурой в области низкая температура.Чтобы использовать ток, два полупроводника нужно. Один n-тип (отрицательный) и один p-тип (положительный), которые образуют термоэлектрическая пара для обеспечения постоянного тока, с которым мощность может производиться, как видно на рис. 2. Они должны быть настроены последовательно электрически, но термически параллельно. [2] Чем больше разность температур, тем больше будет соответствующее напряжение быть, и чем больше способность тока питать нагрузку (т.е. ваш Apple Watch), эффективно преобразуя тепловую энергию в электрическую. энергия.

    Более прочные термоэлектрические генераторы (ТЭГ), используется в крупных промышленных приложениях, таких как газопроводы, сталь литейные заводы и теплоэлектроцентрали имеют возможность генерируют гораздо большую разницу тепла и, следовательно, гораздо большее напряжение. [3] Благодаря большему размеру и доступному пространству многие блоки могут быть связаны между собой. последовательно, обеспечивая большую выходную мощность.Термоэлектрические устройства сбор тепла тела, однако, должен быть намного меньше, гораздо более гибким, и не может ездить с такой большой разницей температур. Эти факторы приводит к гораздо более низкой выходной мощности в диапазоне от микроватт до милливатт. Эти ограничения в сторону, увеличивая добротность или значение Z, что представляет собой отношение эффективности между тепловыми свойствами и электрические свойства используемых материалов, приводы большей мощности выход. [4] Достижения в области нанотехнологий были достигнуты с целью манипулирования структура материалов для создания более высокого значения Z, хотя эти процессы чрезвычайно сложны, и есть ограничения на то, сколько увеличение значения Z, которое они могут создать.

    Нанотехнологии в приложении

    Наше главное внимание при разработке материалов с как можно больше значения Z — это выяснить, как уменьшить теплопроводность. [5] Здесь в игру вступают нанотехнологии. Используя углеродные нанотрубки и графен, исследователи могут разработать наноструктура меньше длины волны света. Это значит, что наблюдается усиление рассеяния фотонов, что приводит к снижение теплопроводности.[5] С электропроводностью все еще на месте, значение Z становится больше, делая материал более эффективен в термоэлектрическом генераторе. Это особенно ценно в сборе тепла тела, так как более высокое значение Z для гибких наноматериалов позволяет значительно повысить преобразование тепла тела в электрическую энергию. Таким образом, эти устройства могут незначительно компенсировать меньшие разность температур при производстве большего напряжения. На На данный момент эти наноструктуры оказались недостаточно эффективными, чтобы их можно было выпускается серийно.При дальнейших исследованиях мы можем ожидать, что это в какой-то степени измениться.

    Будущее

    В более широком смысле термоэлектрические устройства позволяют нам улавливать потраченную впустую энергию за счет тепла, которое часто является побочным продуктом других методы производства энергии. На рынке носимых устройств это означает, что мы может использовать тепловую энергию нашего тела, чтобы обеспечить безграничную мощность некоторые небольшие устройства, если окружающая среда достаточно холодная. Однако существует крайне предельное значение выходной мощности, которое может на самом деле быть реализованным, и это обусловлено тремя факторами.Во-первых, количество тепла, фактически теряемого человеческим телом. Во-вторых, способность проехать достаточно большой dT / T. Наконец, способность использовать это dT / T, так как мы можем только безопасно и практически захватить небольшой процент тепла человеческого тела. Таким образом, общая выходная мощность проект, который будет использоваться всем человечеством (если бы все обладать устройством), даже не повлияет на общую глобальную энергетику требования. Таким образом, как крупномасштабное энергетическое решение, это не выполнимо, но как нишевое приложение интригует.

    и копия Мэтью Стивенса. Автор дает разрешение копировать, распространять и демонстрировать эту работу в неизменном виде, с ссылка на автора, только в некоммерческих целях. Все остальные права, в том числе коммерческие, принадлежат автору.

    Список литературы

    [1] Д. Бехштейн, «Мир Food Budget, Physics 240, Стэнфордский университет, осень 2013 г.

    [2] M. Highland eet al., «Носимый Термоэлектрические генераторы для сбора тепла человеческого тела, Appl. Energy 182 , 518 (2016).

    [3] Ф. ДиСальво, «Термоэлектрическое охлаждение и производство электроэнергии», Science, 285 , 5428 (1999).

    [4] В. Леонов, «Сбор термоэлектрической энергии Тепло человеческого тела для носимых датчиков «, IEEE Sens. J. 13 , 2284 (2013).

    [5] G. Pennelli et al. , «Обзор наноструктурированных устройств для термоэлектрических применений», Байльштейн Дж.нанотехнологий, 5 , 1268-1284, (2014).

    Эффект нагрева от тока — введение, объяснение, применения и решенные примеры

    Как мы все знаем, согласно закону сохранения энергии, энергия не может быть ни создана, ни уничтожена. В нашей повседневной жизни мы замечаем, что все электрическое оборудование, используемое в нашем доме, использует одно и то же электричество. То же электричество, которое используется для вращающихся вентиляторов, также используется для нагрева нашего электрического тостера и т. Д.

    Мы всегда бессознательно наблюдаем нагревательный эффект электрического тока в повседневной жизни. Мы наблюдаем, что всякий раз, когда включается электрическая лампочка, иногда она становится горячей, а когда мы заряжаем свои мобильные телефоны, мы в конечном итоге чувствуем тепловые колебания. Почему все это происходит? Это то, что мы говорим о нагревательных эффектах электрического тока.

    Потери электроэнергии в любом устройстве выражаются в тепловой энергии. Когда электрическая энергия преобразуется в тепловую, это называется тепловым эффектом электрического тока.

    Объясните нагревательный эффект электрического тока

    Всякий раз, когда электрический ток проходит через материалы, он выделяет энергию в виде тепловой энергии. Это преобразование электрической энергии в тепловую, известное как эффект нагрева электрическим током. Хотя это потеря энергии, эта потеря энергии используется для определенных полезных вещей, например, мы используем железные ящики для глажки одежды, электрические нагреватели для кипячения воды и т. Д.

    Нагревательный эффект электрического тока хорошо объясняется с помощью математическое описание, данное законом Джоуля.Закон Джоуля гласит, что количество выделяемого тепла прямо пропорционально току, протекающему через провод, и сопротивлению материала.

    Нагревательный эффект электрического тока определяется законом Джоуля. Математически закон Джоуля определяется выражением:

    => H = I2Rt

    Где,

    H — количество произведенного тепла

    I — количество электричества, проходящего через провод

    t — время, затраченное на выработку тепла

    Применение нагревающего эффекта тока

    1. Электрическая лампочка — одно из основных приложений нагревающего воздействия электричества.Используемая вольфрамовая нить будет выделять свою энергию в виде тепла и света.

    2. Эффект хорошо используется в ящиках с электрическим утюгом.

    3. Электронагреватели — наиболее широко применяемое домашнее оборудование для кипячения воды.

    Решенные примеры

    Вопрос: Медные и нихромовые провода соединены последовательно. В каком случае возможность производства тепла будет больше?

    Ответ: Согласно закону Джоуля количество выделяемого тепла будет прямо пропорционально сопротивлению используемого материала.Среди меди и нихрома сопротивление нихромовой проволоки больше, чем у меди. Следовательно, в случае нихромовой проволоки выделяется больше тепла.

    Формула расхода тепла

    Количество тепла, которое передается в единицу времени в некотором материале.

    Скорость теплового потока в стержне из материала пропорциональна площади поперечного сечения стержня и разнице температур между концами и обратно пропорциональна длине.

    Тепловой поток = — (коэффициент теплопередачи) * (площадь тела) * (изменение температуры) / (длина материала)

    Уравнение:

    Q = -k (А / л) (ΔT)

    У нас:

    Q: теплопередача в единицу времени

    K: теплопроводность

    A: площадь излучающего тела

    л: длина материала.

    ΔT: Разница температур.

    Вопросы по формуле теплопередачи:

    1) Стена дома шириной 7 м и высотой 6 м изготовлена ​​из кирпича толщиной 0,3 м и k = 0,6 Вт / м · К. Температура внутри стены составляет 16 ° C, а снаружи — 6 ° C. Найдите тепловой поток.

    Ответ:

    Разница температур ΔT = T i — T O = 16 ° C — 6 ° C = 10 ° C = 283 K.

    Тепловой поток определяется по формуле:

    Q = -k (А / л) (ΔT)

    Подставляя значения коэффициента теплопроводности, площади, длины и разницы температур внутри и снаружи,

    Q = -0.6 Вт / м · K (7 м * 6 м / 0,3 м) (283 K) =

    Q = -840 Вт

    2) Для подачи нагретой воды используется медная труба диаметром 20 мм, внешняя поверхность трубы имеет k = 6 Вт / м · К, ее толщина 2 мм.

Разное

Добавить комментарий

Ваш адрес email не будет опубликован. Обязательные поля помечены *